NUR775

¡Supera tus tareas y exámenes ahora con Quizwiz!

Recent literature on the long term use of Depo Provera has found that the concern over this topic may not be as much of a concern as previously thought but clinicians should still be aware of this major concern with the prolonged use of progesterone only contraception: A. Increased infection B. Loss of bone density C. Amenorrhea D. Delayed confirmation of pregnancy

B. Loss of bone density

A 30 year old female presents to the clinic with complaints of fishy discharge per vagina. On examination, vagina is not erythematous and cervix looks normal. Which of the following criteria is NOT required for the diagnosis of bacterial vaginosis? A. Predominance of anaerobes B. Plenty of lactobacilli C. Vaginal pH > 4.5 D. Whiff's test positive E. True clue cells on a smear

B. Plenty of lactobacilli

A woman who has used emergency contraception pills should be advised that if she does not have a normal menstrual period within _____ weeks, a pregnancy test should be obtained. A. 1 to 2 B. 2 to 3 C. 3 to 4 D. 4 to 5

C. 3 to 4

A 24-year-old female with multiple sex partners presented with vaginal discharge which is thin and white with fishy odor. Whiff test is positive which indicates the infection with which of the following organism? A. Chlamydia trachomatis B. Trichomonas vaginalis C. Bacterial vaginosis D. Human papilloma virus E. Human immunodeficiency virus

C. Bacterial vaginosis

The clinical syndrome resulting from replacement of normal vaginal flora with anaerobic bacteria is: A. Candida B. Vaginitis C. Bacterial vaginosis D. PID E. Herpes

C. Bacterial vaginosis

Dorothy is a 45-year-old female who complains of a mass in her left breast, dull nipple pain, tenderness of the left nipple, and pasty left nipple discharge. Which of the following conditions should the clinician be most suspicious for? A. Intraductal papilloma B. Hamartomas C. Duct ectasia D. Fibroadenoma

C. Duct ectasia

Three of the following are properly used to treat osteoporosis after menopause. Which one is not? A. Avoidance of corticosteroids. B. Performance of weight-bearing activities for 40 minutes five days a week. C. Estrogen replacement therapy. D. Adequate calcium and vitamin D intake

C. Estrogen replacement therapy.

Beth is breast feeding her 3 month old infant without supplementation. She says she has heard that she cannot get pregnant during this time. Your response to her is: A. It is likely you may become pregnant so you need to use birth control. B. Yes, you are safe as long as you breastfeed. C. For the first six months if you breastfeed and have very little supplementation your chances of getting pregnant are less than 2% D. You are at more risk of getting pregnant now because of your fluctuating hormones.

C. For the first six months if you breastfeed and have very little supplementation your chances of getting pregnant are less than 2%

Which organism is not classically associated with bacterial vaginosis? A. Bacteroides species B. Gardnerella vaginalis C. Histoplasma capsulatum D. Mobiluncus species E. Mycoplasma hominis

C. Histoplasma capsulatum Histoplasma capsulatum is a fungal infection. The other four options are the classical causes of bacterial vaginosis.

Which of the following would be a contraindication for using a diaphragm for birth control? A. Heavy, painful menses B. History of PID C. History of recurrent urinary tract infections D. Multiple pregnancies

C. History of recurrent urinary tract infections

Which of the following medications given to a nursing mother can cause a reduction in her milk supply? A. Antihistamines B. Antithyroid medication C. Oral contraceptives D. Laxatives

C. Oral contraceptives

In a normal menstrual cycle which of the following is produced primarily by the corpus luteum? A. Prolactin B. Estrogen C. Progesterone D. Testosterone

C. Progesterone

Wyonna has been breast feeding for the past three months. She reports to you that she has had no menstrual period since her delivery. She is starting to worry about getting pregnant but wants to continue breast feeding. Of the following which would be the best form of contraception for this client? A. Oral contraceptive pills B. NuvaRing C. Progesterone only contraceptive pills D. Ortho Evra patch

C. Progesterone only contraceptive pills

All of the following are examples of excessive androgens except: A. Increased libido B. Hirsutism C. Syncope D. Acne and oily skin E. Weight gain and edema

C. Syncope

Microscopic image of vaginal discharge shows structures that look like branches A. bacteria B. sperm C. hyphae D. candida

C. hyphae

Which of the following is not a risk factor for bacterial vaginosis (BV)? A. Routine douching B. Receptive oral sex C. Initiating sex at an early age D. Circumcised partner E. Bubble baths

D. Circumcised partner

A 45-year-old woman is seen in the clinic with abnormal uterine bleeding and pain during intercourse. The clinician should consider which of the following diagnoses? A. Postmenopausal syndrome B. Infertility C. Mittelschmerz D. Polyp

D. Polyp

Regarding bacterial vaginosis, which of the following statements is FALSE? A. Referred to by many as nonspecific vaginitis B. the offending organism has been referred to as Haemophilus, Corynebacterium, and Gardnerella C. Associated with characteristic diagnostic findings of clue cells and a vaginal pH of 5.0 to 6.0 D. Treated equally effectively by metronidazole, ampicillin, tetracyclines, or sulfas E. Characterized by a fishy odor

D. Treated equally effectively by metronidazole, ampicillin, tetracyclines, or sulfas

Of all the following contraceptive methods which has the least true life efficacy A. Oral contraceptives B. IUD C. Condoms alone D. Vaginal contraceptive gel

D. Vaginal contraceptive gel Vaginal contraceptive gels (spermacides) should be utilized in conjunction with things like the sponge, a condom, or a diaphragm.

A 22-year-old female presents to her OBGYN with complaints of an increase in vaginal discharge. She describes the discharge as thin and yellow with a strong "fishy" odor. She has been in a monogamous relationship for the last several years. Which of the following findings would you most likely expect on exam? A. KOH prep with branching hyphae and spores B. Negative whiff test C. Strawberry cervix D. Wet prep with clue cells E. Wet prep with flagellated protozoa

D. Wet prep with clue cells

A woman taking estradiol is at risk for developing endometrial cancer.Select one: True False

True

A young female client has received chemotherapeutic medications and asks about any effects the treatments will have related to her sexual health. The most appropriate statement by the nurse is a) "You will experience menopause now." b) "You will need to practice birth control measures." c) "You will continue having your menses every month." d) "You will be unable to have children."

b) "You will need to practice birth control measures."

The Nurse Practitioner is providing teaching to a postpartum Catholic client who wishes to use natural family planning. Which of the following would the NP include in teaching about how to utilize this method?Select all that apply. A. Calendar charting B. Cervical mucous observation C. Extended breastfeeding D. Basal body temperature graphing E. The use of condoms with no spermicide

A. Calendar charting B. Cervical mucous observation C. Extended breastfeeding D. Basal body temperature graphing

Which of the following contraceptive options are progestin-only? Choose three answers.Select all that apply. A. Depo-Provera B. Mirena intrauterine device C. NuvaRing D. Implanon E. Paraguard intrauterine device

A. Depo-Provera B. Mirena intrauterine device D. Implanon

Which of the following is false regarding usage of the Ortho Evra patch? A. If the patch falls off and stays off for greater than 12 hours, restart a new four-week cycle and use a backup method of protection. B. After three applications, the patch is removed for seven days, allowing for menstruation. C. The patch may be applied to the arm, buttocks, or abdomen. D. The patch is removed every seven days and another one is applied.

A. If the patch falls off and stays off for greater than 12 hours, restart a new four-week cycle and use a backup method of protection.

Your patient is worried about which contraceptive option would be best for her. She states that she often forgets to take her vitamins. For this particular candidate, you would discuss which of the following?Select all that apply. A. NuvaRing B. Depot medroxyprogesterone C. Progestin-only pills D. An intrauterine device

A. NuvaRing B. Depot medroxyprogesterone D. An intrauterine device

Your patient asks you to prescribe her the most effective method of contraception. Which method has the highest failure rate? A. Spermicides B. Diaphragm C. Oral contraceptives D. Depo-Provera

A. Spermicides

In contrast to progestin-only emergency contraception, a possible mechanism of action of ulipristal (Ella) is: A. inhibiting embryo implantation B. impairing sperm transport C. through spontaneous abortion D. impairing ovum transport

A. inhibiting embryo implantation

In contrast to progestin-only emergency contraception, a possible mechanism of action of ulipristal (Ella) is: A. inhibiting embryo implantation B. impairing sperm transport. C. through spontaneous abortion. D. impairing ovum transport

A. inhibiting embryo implantation

A 22-year-old woman taking a 35-mcg ethinyl estradiol COC calls after forgetting to take her pills for 2 consecutive days. She is 2 weeks into the pack.You advise her to: A. take the last pill missed immediately, even if this means taking 2 pills today. B. discard two pills and take two pills today. C. discard the rest of the pack and start a new pack with the first day of her next menses. D. continue taking one pill daily for the rest of the cycle.

A. take the last pill missed immediately, even if this means taking 2 pills today.

Which of the following is commonly found after 1 year of using DMPA (Depo-Provera)? A. weight gain. B. hypermenorrhea. C. acne. D. rapid return of fertility when discontinued

A. weight gain.

Due to an increased risk of blood clots, an alternative to the contraceptive ring (NuvaRing) or patch (Ortho Evra) is preferred in all of the following women except: A. a 42-year-old nulliparous woman B. 31-year-old woman with a history of naturally occurring multiple gestation pregnancy. C. 28-year-old who smokes one pack per day D. 33-year-old woman with a family history of venous thrombosis.

B. 31-year-old woman with a history of naturally occurring multiple gestation pregnancy.

Due to an increased risk of blood clots, an alternative to the contraceptive ring (NuvaRing) or patch (Ortho Evra) is preferred in all of the following women except: A. a 42-year-old nulliparous woman B. 31-year-old woman with history of naturally occurring multiple gestation pregnancy. C. 28-year-old who smokes one pack per day D. 33-year-old woman with a family history of venous thrombosis.

B. 31-year-old woman with history of naturally occurring multiple gestation pregnancy.

According to the U. S. Medical Eligibility Criteria for Contraception Use, which of the following is a clinical condition in which use of a copper-containing IUD should be approached with caution? A. uncomplicated valvular heart disease B. AIDS-defining illness C. hypertension D. dysmenorrhea

B. AIDS-defining illness

Your patient is currently menstruating and forgot to take her combined contraceptives yesterday. You advise her A.Discard the pack and start a new park B. If she missed a non active pill, she should resume the pills as scheduled the next day C. Resume her pills as scheduled tomorrow D. Take two pills today and get on schedule tomorrow by taking one again

B. If she missed a non active pill, she should resume the pills as scheduled the next day

All of the following are examples of excessive progesterone except: A. Increased appetite B. Increased libido C. Breast tenderness D. Depression

B. Increased libido

When counseling a patient on the risks associated with an intrauterine device, you include all of the following except: A. Intrauterine device strings may migrate due to poor positioning and become difficult to find B. Intrauterine device use increases the risk for pelvic inflammatory disease C. Intrauterine device use increases the risk for candidiasis D. Risk of expulsion of the intrauterine device is 10% the first year of use E. All of the answer choices are true

B. Intrauterine device use increases the risk for pelvic inflammatory disease

26-year-old mother who breastfeeds her 10-month-old child queries about contraceptives. In counseling her on the use of the progestin-only pill (POP), you mention all of the following except: A. the pill is taken every day B. POP is a more effective contraceptive than COC C. POP does not alter the quality or quantity of breast milk. D. POP is associated with bleeding irregularity, ranging from prolonged flow to amenorrhea.

B. POP is a more effective contraceptive than COC

A 26-year-old mother who breastfeeds her 10-month-old child queries about contraceptives. In counseling her on the use of the progestin-only pill (POP), you mention all of the following except: A. the pill is taken every day B. POP is a more effective contraceptive than COC C. POP does not alter the quality or quantity of breast milk. D. POP is associated with bleeding irregularity, ranging from prolonged flow to amenorrhea.

B. POP is a more effective contraceptive than COC

Which of the following statements should the Nurse Practitioner include in medication teaching about oral contraceptives? A. Progestin-only pills are as effective as the combined pill with typical use. B. Progestin-only pills may be less effective than the combined pill. C. Progestin-only contraceptive pills thicken the endocervical mucus, alter the endometrium, and suppress ovulation. D. Benefits of combined oral contraceptives may include decreased menstrual cramps and PMS. E. Benefits of progestin-only pills include a daily schedule that's easy to remember and less nausea than with combined oral contraceptives

B. Progestin-only pills may be less effective than the combined pill. C. Progestin-only contraceptive pills thicken the endocervical mucus, alter the endometrium, and suppress ovulation. D. Benefits of combined oral contraceptives may include decreased menstrual cramps and PMS.

Which of the following is true about therapy with Depo-Provera? A. It is not appropriate for adolescents. B. Return of fertility may be delayed after discontinuing the drug. C. It may be prescribed up until age 35. D. It may be prescribed only for 2 years due to decreased bone mineral density.

B. Return of fertility may be delayed after discontinuing the drug

Your 30-year-old patient has been taking oral contraceptives for over ten years. She has recently stopped taking them and wonders how quickly she can safely conceive. How do you respond? A. She may conceive after one month off of the oral contraceptives. B. She can safely conceive immediately. C. She may have over one year to return to safe fertility. D. She will only be able to safely conceive after three months off of the hormones.

B. She can safely conceive immediately.

The nurse practitioner receives a phone call from an 18-year-old female patient on combined oral contraceptive pills, stating that she missed the previous two days of her birth control pill. She is inquiring as to what she should do now, as these are the "active pills". The NP advises her to: A. Take two active pills today and two active pills tomorrow, then resume the regular schedule. B. Take the most recent missed active pill as soon as possible, discard other missed pills, and continue with the regular schedule, even if it means taking two pills in one day. Use backup contraception or avoid intercourse for 7 days. C. Discard the current pack and begin with a new one the following day. D. Resume taking the pills as scheduled the following day.

B. Take the most recent missed active pill as soon as possible, discard other missed pills, and continue with the regular schedule, even if it means taking two pills in one day. Use backup contraception or avoid intercourse for 7 days.

A college freshman who is using oral contraceptives calls the nurse practitioner's office asking for advice. She forgot to take her pills 2 days in a row during the second week of the pill cycle and wants to know what to do. What is the best advice? A. Start a new pack of pills and dispose of the old one B. Take two pills today and two pills tomorrow, and have your partner use condoms for the rest of the pill cycle C. Stop taking the pills right away, and start a new pill cycle in 2 weeks D. Take one pill today and two pills tomorrow, and have your partner use

B. Take two pills today and two pills tomorrow, and have your partner use condoms for the rest of the pill cycle

Which of the following is not an absolute contraindication for use of oral contraceptive pills? A. Active hepatitis A infection B. Thrombosis related to an IV needle C. Undiagnosed vaginal bleeding D. Transient ischemic attack (TIA)

B. Thrombosis related to an IV needle

The reduction in free androgens noted in a woman using COC can yield an improvement in: A. cycle control B. acne vulgaris C. breast tenderness. D. rheumatoid arthritis

B. acne vulgaris

When prescribing the contraceptive patch (Ortho Evra) or vaginal ring (NuvaRing), the NP considers that: A. these are progestin-only products. B. candidates include women who have difficulty remembering to take a daily pill. C. there is significant drug interactions with both products D. contraceptive efficacy is less than with COC.

B. candidates include women who have difficulty remembering to take a daily pill.

Irregular bleeding associated with DMPA (Depo-Provera) can be minimized with the use of all of the following except: A. acetaminophen. B. ibuprofen. C. naproxen sodium. D. estrogen supplements.

B. ibuprofen.

The most common reasons for discontinuing oral contraception use is breakthrough bleeding and: A. nausea/vomiting B. inconvenience of use. C. cost D. high failure rate.

B. inconvenience of use.

With DMPA in depot injection (Depo-Provera), the recommended length of use is usually: A. less than 1 year. B. no more than 2 years. C. as long as the woman desires this form of contraception. D. as determined by her lipid response to the medication.

B. no more than 2 years.

A 23-year-old woman is breastfeeding her healthy newborn. She wishes to start using hormonal contraception. Which of the following represents the best regimen? A. combined oral contraception initiated at 2 weeks B. progesterone-only oral contraception initiated at 3 weeks C. medroxyprogesterone acetate (Depo-Provera) given day 1 postpartum D. use of all forms of hormonal contraception is discouraged during lactation

B. progesterone-only oral contraception initiated at 3 weeks

You see a 34-year-old woman who reports having unprotected sexual intercourse 4 days ago and requests emergency contraception. She has a recent history of gonorrhea that was treated successfully. The most acceptable and effective option in this clinical scenario is: A. progestin-only emergency contraception B. ulipristal C. copper-containing IUD D. nothing, as 4 days is too long for emergency contraception to be effective

B. ulipristal

By using a diaphragm with spermicide nonoxynol-9 during sexual intercourse, a woman is likely at increased risk for: A. cervical stenosis B. urinary tract infection C. increased perivaginal lactobacilli colonization D. ovarian malignancy.

B. urinary tract infection

As you prescribe COC containing the progestin drospirenone (Loryna, Ocella, Vestura, Yasmin, Yaz), you offer the following advice: A. "Always take this pill on a full stomach. " B. "You should not take acetaminophen when using this birth control pill. " C. "Avoid using potassium-containing salt substitutes." D. "You will likely notice that premenstrual syndrome symptoms might become worse.

C. "Avoid using potassium-containing salt substitutes."

Which of the following is the most appropriate response to a 27-year-old woman who is taking phenytoin (Dilantin) for the treatment of a seizure disorder and is requesting hormonal contraception? A. "A barrier method would be the preferable choice." B. "COC is the best option." C. "Depo-Provera (medroxyprogesterone acetate in a depot injection [DMPA]) use will likely not interact with your seizure medication." D. "Copper-containing IUD use is contraindicated."

C. "Depo-Provera (medroxyprogesterone acetate in a depot injection [DMPA]) use will likely not interact with your seizure medication."

A woman who has used emergency contraception pills should be advised that if she does not have a normal menstrual period within _____ weeks, a pregnancy test should be obtained. A. 1 to 2 B. 2 to 3 C. 3 to 4 D. 4 to 5

C. 3 to 4

At what point after childbirth can a combined oral contraceptive be started without other risk factors for venous thrombosis in a woman who is not breastfeeding? A. 1 day B. 1 week C. 3 weeks D. 6 weeks

C. 3 weeks

Which of the following statements is true concerning vaginal diaphragm use? A. When in place, the woman is aware that the diaphragm fits snugly against the vaginal walls. B. This is a suitable form of contraception for women with recurrent urinary tract infection. C. After insertion, the cervix should be smoothly covered. D. The device should be removed within 2 hours of coitus to minimize the risk of infection.

C. After insertion, the cervix should be smoothly covered.

The following conditions are absolute contraindications for the use of oral contraceptives except: A. Sexually active patient with amenorrhea B. History of emboli that resolved with heparin therapy 15 years ago C. Cigarette smoking at the age of 30 years D. Hepatitis C infection

C. Cigarette smoking at the age of 30 years

When assessing the need for emergency contraception. Your patient tells you it is been 5 days since she had unprotected sex. What form of emergency contraception would be indicated for this patient? A. Oral contraception - Levonorgestrel (Plan B) B. Depo-Provera (DMPA) C. Copper-releasing Intrauterine Device (IUD) D. Spermicide Douche

C. Copper-releasing Intrauterine Device (IUD)

A patient visits the clinic with complaints of oral contraceptive side effects. Which one of the following is NOT a sign of excessive estrogen? A. Breast tenderness B. Increased blood pressure C. Decreased blood pressure D. Headache E. Melasma

C. Decreased blood pressure

Which statement is false regarding the usage of NuvaRing? A. It is vaginally inserted. B. It is left in place for 21 days and then removed to allow for menstruation. C. If the ring falls out, it should be reinserted within 24 hours. D. It is a combination of synthetic estrogen and progestin.

C. If the ring falls out, it should be reinserted within 24 hours.

A 26-year-old female comes to your office to discuss birth control options. Her history includes migraine headaches with aura while on combination oral contraceptives in the past. She does not want to become pregnant. Which of the following birth control options would be the best choice for her? A. Combined hormonal contraceptive pills. B. Ortho Evra Patch C. Mirena intrauterine device (IUD) D. Vaginal NuvaRing

C. Mirena intrauterine device (IUD)

The incidence of which of the following cancers are reduced with the use of oral contraceptives? A. Ovarian and breast B. Endometrial and breast C. Ovarian and endometrial D. Lung and ovarian

C. Ovarian and endometrial

All of the following are correct statements regarding oral contraceptives except: A. The actual failure rate of oral contraceptives is 3% B. Desogestrel belongs to the progesterone family of drugs C. The newer low-dose birth control pills do not require backup during the first 2 weeks of use D. Oral contraceptives are contraindicated for women 35 years of age or older who smoke

C. The newer low-dose birth control pills do not require backup during the first 2 weeks of use ???

A 19-year-old student who is on a prescription of combined oral contraceptive pills is being seen for an annual gynecological exam in the college health center. The nurse practitioner has obtained the Pap smear and is about to perform the bimanual exam. She gently removes the plastic speculum from the vagina. While the NP is performing the bimanual vaginal exam, the patient complains of slight discomfort during deep palpation of the ovaries. Which of the following is a true statement? A. The uterus and the ovaries are both very sensitive to any type of palpation B. The fallopian tubes and ovaries are not sensitive to light or deep palpation C. The ovaries are sensitive to deep palpation but they should not be painful D. The uterus and the ovaries are not important organs of reproduction

C. The ovaries are sensitive to deep palpation but they should not be painful

Which of the following women is the best candidate for progestin-only pill (POP) use? A. an 18-year-old woman who frequently forgets to take prescribed medications B. a 28-year-old woman with multiple sexual partners C. a 32-year-old woman with adequately-controlled hypertension D. a 26-year-old woman who wants to use the pill to help "regulate" her menstrual cycle

C. a 32-year-old woman with adequately-controlled hypertension

Which of these emergency contraceptives are contraindicated in migraine and CVD? A. Copper IUD B. levonogestrel C. ethinyl estradiol and levonogestrel combination D. all are acceptable E. none are acceptable

C. ethinyl estradiol and levonogestrel combination

Guidelines recommend that a breastfeeding mother waits until breastfeeding is well established for approximately 6 months before using combined oral contraceptives (COC) because: A. in early breastfeeding, the amount of hormone in COC could cause significant harm to the nursing infant. B. efficacy of COC could be compromised by breastfeeding. C. milk flow could be compromised by COC. D. COC oral contraceptive use could affect mother's sleep patterns.

C. milk flow could be compromised by COC.

A 25-year-old female patient has been using Depo-Provera, but has experienced an 8-lb weight gain in the past year and is already significantly overweight. She asks if the weight gain is common. How should the NP respond? A. "This is, unfortunately, common with the use of Depo-Provera. If your weight has normalized and this is tolerable for you, you can continue it." B. "This is an abnormal side effect associated with the use of this drug, but if this weight is tolerable for you, you can continue it." C. "This is a very abnormal side effect associated with the use of this drug." The NP should advise that she switch to a different form of contraception. D. "This is, unfortunately, common with the use of Depo-Provera." As she is already overweight, the NP should offer a switch to a different form of contraception like the nonhormone intrauterine device.

D. "This is, unfortunately, common with the use of Depo-Provera." As she is already overweight, the NP should offer a switch to a different form of contraception like the nonhormone intrauterine device.

In regards to oral hormonal contraceptives, all are false except? A. Benefits may include minimizing the risk of breast cancer B. Progestin-only pills are as effective as the combined pill with typical use. C. Actual failure rates are between 0.5% and 1% D. Benefits may include reduced dysmenorrhea

D. Benefits may include reduced dysmenorrhea

Your patient is currently menstruating and forgot to take her combined oral contraceptive pill yesterday. She calls the clinic. You advise her of which of the following? A. Resume taking her pills as scheduled again tomorrow B. Take two today and get back on schedule tomorrow by taking one again C. Discard this pack and start a new pack D. If she missed a nonactive, nonhormonal pill, that is okay. She should resume taking her pills as scheduled again tomorrow.

D. If she missed a nonactive, nonhormonal pill, that is okay. She should resume taking her pills as scheduled again tomorrow.

Which is incorrect regarding the use of a vaginal diaphragm? A. All of the answer choices are true B. It may increase the risk for urinary tract infections C. It must be left in place for at least six hours after intercourse D. It should be refitted if one has a weight gain of greater than 10 lbs

D. It should be refitted if one has a weight gain of greater than 10 lbs

Concerning selective estrogen receptor modulator therapy such as raloxifene (Evista), which of the following statements is correct? A. Concurrent progestin opposition is needed B. Hot flashes are reduced in frequency and severity. C. Use is contraindicated when a woman has a history of breast cancer D. Osteoporosis risk is reduced with use.

D. Osteoporosis risk is reduced with use.

Your 27-year-old patient who is suffering from chronic migraine headaches visits your clinic for contraceptives. She is otherwise healthy and does not smoke. You advise that the best option for her is: A. Ortho Evra patch B. NuvaRing C. Combined estrogen and progestin pills D. Progestin-only pills

D. Progestin-only pills

Your 32-year-old patient has been using Depo-Provera and recently received her shot before getting married. She would like to start planning to have a family. She visits your clinic hoping to get pregnant within the next three months. Your response to her is that: A. She should have an immediate return to fertility. B. She will have an immediate return to fertility after three months since she just received a shot. C. She will have an immediate return to fertility as soon as her period is regular again. D. She may have a delay in her return to fertility for up to one year or longer.

D. She may have a delay in her return to fertility for up to one year or longer.

Which of the following factors is considered a relative contraindication for combined oral contraceptive pills? A. Undiagnosed vaginal bleeding B. Hepatoma of the liver C. Suspected history of transient ischemic attacks D. Smoking

D. Smoking

Emergency contraception refers to: A. An induced abortion in an emergency room (ER). B. Quickly starting on birth control pills in anticipation of sexual intercourse. C. Having a medroxyprogesterone (Depo Provera) injection in the ER every 12 weeks. D. Taking emergency contraceptive pills.

D. Taking emergency contraceptive pills

Which of the following statements is false? A. Progestin-only emergency contraception can be taken as one dose or two doses. B. Ulipristal is available by prescription only C. Progestin-only emergency contraception is available OTC for women 17 years old and older D. Ulipristal is taken in two doses 12 hours apart.

D. Ulipristal is taken in two doses 12 hours apart.

An 18-year-old woman requests emergency contraception after having unprotected vaginal intercourse approximately 18 hours ago. Today is day 12 of her normally 27- to 29-day menstrual cycle and she has no contraindications to the use of any currently available forms of emergency contraception. You advise her that: A. emergency hormonal contraception use reduces the risk of pregnancy by approximately 33%. B. all forms of emergency contraception must be used within 12 hours after unprotected intercourse. C. the likelihood of conception is minimal. D. insertion of a copper-containing IUD offers an effective form of emergency and ongoing contraception.

D. insertion of a copper-containing IUD offers an effective form of emergency and ongoing contraception.

A 24-year-old woman who requests emergency contraception pills wants to know the effects if pregnancy does occur. You respond that there is the risk of increased rate of: A. spontaneous abortion B. birth defects C. placental abruption D. none of the above.

D. none of the above.

When counseling a woman about COC use, you advise that: A. long-term use of COC is discouraged because the body needs a "rest" from birth control pills from time to time. B. fertility is often delayed for many months after discontinuation of COC. C. there is an increase in the rate of breast cancer after protracted use of COC D. premenstrual syndrome symptoms are often improved with use of COC.

D. premenstrual syndrome symptoms are often improved with use of COC.

With the use of a levonorgestrel intrauterine system (Mirena), which one of the following is normally noted? A. endometrial hyperplasia B. hypermenorrhea C. increase in PID rates D. reduction in menstrual flow

D. reduction in menstrual flow

A 38-year-old nulliparous woman who smokes two and a half packs a day is in an "on-and-off" relationship. The woman presents seeking contraception. Which of the following represents the most appropriate method? A. contraceptive ring (NuvaRing). B. COC. C. contraceptive patch (Ortho Evra). D. vaginal diaphragm

D. vaginal diaphragm

The primary risk factor for development of breast cancer in women of average risk is: a. Age. b. Smoking history. c. Number of live births. d. Exposure to estrogen.

a. Age. a. Age is the most important risk factor for developing breast cancer in women of average risk. Breast cancer is more common in older women and has a higher mortality rate when discovered. Nearly 85% of breast cancer occurs in women who are 50 years of age or older. Exposure to estrogen and genetic factors are important risk factors, but do not contribute to development of breast cancer as greatly as age does in women of average risk. Hence, the reason for diligent annual screening in women who are 50 years of age and older. Hollier, A. (2013) Adult-gero and family nurse practitioner certification practice questions p. 77

A second triple screen on a 35 year-old primigravida reveals abnormally low levels of ath alpha fetoprotein and estriol and high levels of human chorionic gonadotropin. Which of the following interventions is the best choice for this patient? a. Order an ultrasound b. Order a computed tomography (CT) scan of the abdomen c. Order a 24-hour urine for protein clearance d. Assess for a history of illicit drug or alcohol use

a. Order an ultrasound Abnormally low levels of alpha fetoprotein and estriol and high levels of human chorionic gonadotropin are abnormal during pregnancy. An ultrasound should be ordered to further evaluate the fetus for characteristics of Down syndrome and/or fetal demise. Codina-Leik, M. T. (2014) Family nurse practitioner certification intensive review fast facts and practice questions

A diagnosis of osteoporosis can be made when: a. An x-ray indicates fragility fractures. b. Bone mineral density is 2.5 standard deviations below the mean. c. T-score greater than 2.5. d. DXA scan indicates fragility fracture index.

b. Bone mineral density is 2.5 standard deviations below the mean Osteoporosis can be diagnosed with a bone mineral density test (BMD). The technique used is a dual energy x-ray absorptiometry (DXA). This is the most common clinical tool used to diagnose osteoporosis. Osteoporosis can also be diagnosed when there is a fragility fracture. This can be identified on x-ray. In an absence of trauma, a fragility fracture may indicate osteoporosis, multiple myeloma or other diseases. BMD that is 2.5 or more standard deviations below the young adult mean (or T-score of -2.5 or less) constitutes a diagnosis of osteoporosis. Hollier, A. (2013) Adult-gero and family nurse practitioner certification practice questions p.75

A 16 year-old female is diagnosed with primary dysmenorrhea. She has taken over the counter ibuprofen in 800 mg increments every 8 hours during menses for the past 3 months with minimal relief of symptoms. What intervention will provide greatest relief of dysmenorrhea symptoms? a. Flurbiprofen during menses b. Combined oral contraceptives c. Daily multivitamin with B12 supplementation d. 30 minutes of regular exercise daily

b. Combined oral contraceptives NSAIDs and hormonal contraceptives represent the mainstay of pharmacologic treatment for dysmenorrhea. NSAIDs produce an 80-86% response rate when used for dysmenorrhea. The general recommendation is that when one agent (NSAIDs or hormonal contraceptives) does not produce relief of symptoms, the other agent should be tried. Hence, the best choice is oral contraceptives. Both agents should be considered for women who are symptomatic with one agent only.

A 51 year old female patient presents with a 2 cm palpable breast mass. How should this be evaluated to determine whether it is solid or cystic in nature? a. mammogram b. ultrasound c. MRI d. clinical breast exam

b. ultrasound

A 21 year-old female presents with three 0.5 cm human papilloma virus (HPV) lesions on her vulva. An appropriate treatment option for this patient would be: a. Acetic acid b. Colposcopy. c. Podophyllin. d. Acyclovir.

c. Podophyllin. This patient has HPV. This is a viral infection that increases a woman's risk of cervical cancer. In males there is an increased risk of cancer of the anus and penis. The warts that are produced are painless and usually appear within weeks of infection. There are several topical treatments for HPV, imiquimod, podophyllin, and trichloroacetic acid (TCA). One of these agents may be applied to the warts. Treatment are generally well tolerated. TCA may cause burning. The warts will slough off after one or more treatments. There are no oral antiviral agents indicated for treatment of HPV. Codina-Leik, M. T. (2014) Family nurse practitioner certification intensive review fast facts and practice questions

A 50 year-old female believe that she is menopausal. She complains of hot flashes and has no had menses in 12 months. Which of the following test results may be helpful for confirmation of menopause? a. Increased thyroid stimulating hormone b. Decreased follicle stimulating hormone c. Hypoestrogenemia d. Increased follicle stimulating hormone

d. Increased follicle stimulating hormone Answer: d. Follicle stimulating hormone (FSH) begins to rise during menopausal transition. This stage of menopause begins with variation in menstrual cycle length and ends 12 months after the final menses. If the patient is older than 45 years and menstrual cycle dysfunction has been ruled out, menopause should be considered. Possibly the best approach to diagnosing menopause is to ask and observe clinical manifestations. An elevated FSH is not necessary to make a diagnosis of menopause but is commonly done in clinical practice. Diagnosis entails a review of her symptoms. Specifically, changes in bleeding patterns, hot flashes, sleep disturbances, and genitourinary symptoms are characteristic of menopause. Hollier, A. (2013) Adult-gero and family nurse practitioner certification practice questions p. 73

A 25-year-old patient presented at the clinic due to profuse vaginal discharge. The presence of heavy concentration of coccobacilli surrounding vaginal epithelial cells with loss of distinct cell margins is the appearance of; A. Donovan bodies B. Clue cells C. Chancre D. Inclusion cells E. Koliocytes

B. Clue cells

Which of the following tests is essential for a 46-year-old woman who the clinician suspects is perimenopausal? A. Pregnancy B. Estrogen level C. Progesterone level D. LH level

A. Pregnancy

A 20-year-old G0P0 woman with no significant medical history presents to your office reporting that she is concerned about becoming pregnant. She had sexual intercourse with her boyfriend last night and usually uses a cervical diaphragm for contraception. However, she forgot to place her diaphragm last night and is now seeking emergency contraception. She also wants to prevent this situation from happening again. Which of the following is the best option for this patient for emergency contraception? A- Implanon B- Copper intrauterine device C- Medroxyprogesterone D- Oral contraceptive pills

B- Copper intrauterine device

One of the nurse practitioner's patients plans on using a diaphragm for contraception. While counseling her on how to use it, she asks when she should remove the diaphragm following intercourse. What would be the most appropriate answer to her question? A. 1 hour following intercourse B. Whenever is most convenient C. 6 hours following intercourse D. Immediately following intercourse

C. 6 hours following intercourse

Noncontraceptive benefits of COC use include a decrease in all of the following except: A. iron-deficiency anemia. B. pelvic inflammatory disease (PID). C. cervicitis. D. ovarian cancer.

C. cervicitis.

Sharon tells you that she has been douching after sex and believes this is an effective birth control option. You should tell her: A. Douching prevents sperm from entering the uterus B. Douching should be done once monthly after menses C. Douching is a reliable form of contraception D. Douching may increase her risk of ectopic pregnancy

D. Douching may increase her risk of ectopic pregnancy

A 47-year-old woman presents with complaints of pain with intercourse, intense itching "down there," and states "it looks different down there." She denies bleeding, foul odor. Which of the following diagnoses should the clinician consider most likely? A. Atrophic vaginitis B. Trichomoniasis C. Candidiasis D. Vulvar lichen sclerosus

D. Vulvar lichen sclerosus

A female who is pregnant and has developed bacterial vaginosis. She is treated and discharged. What is not a complication of a pregnancy associated with bacterial vaginosis? A. Late miscarriage B. Low-birth-weight infant C. Chorioamnionitis D. Postpartum endometritis E. Postpartum blues

E. Postpartum blues

A patient requests HIV screening: Recommendations?

Screened today...repeat in 4-6 weeks...and 3 months

Which of the following types of estrogen is secreted in the greatest amounts during the reproductive years and is considered most potent? A. Estrone (E1) B. Estradiol (E2) C. Estriol (E3) D. Potency and secretion of all the above are in equal amounts

B. Estradiol (E2)

According to the U. S. Medical Eligibility Criteria for Contraception Use, a 25-year-old with hypertension a candidate for a copper-containing IUD True False

True

Sydney, age 21, is taking an oral contraceptive (OC). She complains of acne. How should you adjust the estrogen in the OC? A. Increase the estrogen content. B. Decrease the estrogen content. C. Delete the estrogen content. D. No adjustment should be made to the estrogen content.

A. Increase the estrogen content.

Your patient calls your office and states that she thinks that her NuvaRing "fell out" she asks about reinserting it. You know that the maximal amount of time that the NuvaRing can be outside the body before needing to be discarded for lack of efficacy is: A. 3 hours B. 6 hours C. 12 hours D. 24 hours

A. 3 hours

Vaginal infections that are resulting from an overgrowth of normal flora are called A. Endogenous infections B. Chronic infections C. Iatrogenic infections D. Super infections E. Systemic infections

A. Endogenous infections

When can a woman safely conceive after discontinuing COC use? A. immediately B. after 1 to 2 months C. after 3 to 4 months D. after 5 to 6 months

A. immediately

A 54 year old female presents with a small to moderate amount of vaginal bleeding of recent onset. She has been postmenopausal for approx. 2 years. What diagnosis is least likely? a. endometrial carcinoma b. ovarian cancer c. endometrial hyperplasia d. uterine polyps

b. ovarian cancer

A 24-year-old woman has had an abnormal vaginal discharge for the past week. It is off-white and non-itchy, with an offensive odour. She has had one sexual partner in the last 8 months, and he has no symptoms. There is an off-white vaginal discharge of pH 6.4 pooling in the posterior fornix, with no inflammation of the vulva or vagina. Which is the single most likely finding on a Gram-stained sample of the vaginal discharge? A. Gram-negative intracellular diplococci B. Gram-positive and Gram-negative mixed bacteria C. Numerous lactobacilli D. Polymorphonuclear leucocytes E. Yeast cells with hyphae

B. Gram-positive and Gram-negative mixed bacteria

Your 25 year old G1P1 patient is requesting oral contraceptives. She is normal weight and does not smoke or have any significant health factors. On history she notes that her menses are heavy and she has difficulty with her pregnancy due to nausea. Your exam reveals cystic breasts, but no other physical findings. The best OCP this patient would benefit from would be: A. High estrogenic activity B. Low estrogenic activity C. High progestational activity D. Low progestational activity

B. Low estrogenic activity Excessive estrogen effects include dysmenorrhea, nausea, chloasma (mask like appearance) CVA, DVT, Thromboembolitic disease, Pulmonary emboli, telangiectasia (spider vein appearance on face), Hepatic adenoma/adenocarcinoma, cervical changes, and breast tenderness secondary to increase of the size.

The NP is seeing an adult female patient with 2 school-aged children in a monogamous relationship. She is asking about non-oral contraceptive methods but does not want to consider permanent sterilization for herself or her partner. This patient has no significant medical history but smokes 1 pack of cigarettes daily. Which of the following is the best contraceptive option for this patient? A. Transdermal Contraceptive Patch B. Etonogestrel Subdermal Implant C. Diaphragm D. Levonorgestrel Intrauterine Device (IUD)

D. Levonorgestrel Intrauterine Device (IUD)

All of the following are infections that affect mostly the labia and vagina except: a. Bacterial vaginosis b. Candidiasis c. Trichomoniasis d. Chlamydia trachomatis

d. Chlamydia trachomatis Infections that commonly affect the labial and vagina include bacterial vaginosis, candidiasis, and trichomoniasis. Chlamydia trachomatis commonly affects the cervix, endometrial lining, fallopian tubes, and pelvic cavity. Codina-Leik, M. T. (2014) Family nurse practitioner certification intensive review fast facts and practice questions

A 20-year-old woman is seen in the clinic because her boyfriend was found to have gonorrhea. Which of the following is the treatment of choice for gonorrhea? A. Ceftriaxone B. Doxycycline C. Acyclovir D. Metronidazole

A. Ceftriaxone

An acceptable option for birth control for an obese 25 year old smoker would be: A. Cervical cap B. Ortho Evra patch C. Depo-vera D. Mini pill

A. Cervical cap

Which lab test is important to obtain after a 10 year profile of oral contraceptives have been utilized by a patient: A. Liver profile B. Lipid profile C. CBC with diff D. Metabolic panel

A. Liver profile

A female patient has just been diagnosed with bacterial vaginosis (BV). Which of the following describes the typical symptoms of this disorder? A. Malodorous, thin, "fishy" smelling vaginal discharge B. "Cottage cheese-like" vaginal discharge with vulvar erythema, pruritus C. Green-gray, profuse vaginal discharge with vulvovaginal erythema D. White or clear, non-offensive vaginal discharge E. Pruritus, erythema, dyspareunia

A. Malodorous, thin, "fishy" smelling vaginal discharge

Mildred is 6 months pregnant and presents with symptoms of urinary frequency, urgency, dysuria, and suprapubic discomfort. Her urine is cloudy and malodorous. She has no chills, fever, nausea, or vomiting. What is your diagnosis? A. Urinary calculi B. Acute cystitis C. Acute pyelonephritis D. Interstitial cystitis

B. Acute cystitis Symptoms of urinary frequency, urgency, dysuria, and suprapubic discomfort in the absence of chills, fever, nausea, and vomiting, along with cloudy, malodorous urine, are clinically diagnostic of acute cystitis. Acute cystitis and acute pyelonephritis are common renal disorders in pregnancy. Renal calculi may cause intermittent flank pain or pain that radiates around to the abdomen. Maternal symptoms of pyelonephritis include fever, shaking chills, malaise, flank pain, nausea and vomiting, headache, increased urinary frequency, and dysuria. Interstitial cystitis is a chronic, painful bladder disorder in which the course is unpredictable. The symptoms include urinary frequency, urgency, nocturia, and suprapubic pain in the absence of urinary pathogens. Although the etiology of interstitial cystitis is unknown, most attribute it to an initial insult to the bladder wall by a toxin, allergen, or immunologic agent that causes an inflammatory response.

A 17 y.o. requests to start Depo-Provera injections as her method of birth control. She discloses to you that she has had four sexual partners in the last year. Her last menstrual period was twelve days ago. The appropriate management of this patient would be: A. Administer the Depo-Provera injection today B. Advise her to use another method of birth control now and return when she starts her menses. C. Give her an injection after a negative pregnancy test and advise her to utilize condoms for the next seven days D. Giver her and injection and tell her to use a barrier method for seven days.

B. Advise her to use another method of birth control now and return when she starts her menses. Can be given within 5 days after starting menses and should be repeated every 12 weeks.

Which of the following is true of the IUD (intrauterine device)? A. The IUD is 95% effective at preventing pregnancy. B. The IUD has an inhibitory effect on sperm capacitation. C. The IUD can only be inserted at menses. D. The IUD can only be inserted in women with multiparity.

B. The IUD has an inhibitory effect on sperm capacitation.

A 48-year-old woman presents with a monthly 4-day premenstrual migraine headache, poorly responsive to triptans and analgesics, and accompanied by vasomotor symptoms (hot flashes). The clinician considers prescribing all of the following except: A. continuous monophasic oral contraceptive. B. phasic combined oral contraceptive with a 7-day-per-month withdrawal period. C. low-dose estrogen patch use during the premenstrual week. D. triptan prophylaxis.

B. phasic combined oral contraceptive with a 7-day-per-month withdrawal period.

Margie, who has been breastfeeding for 4 weeks, has mastitis in her right breast and is taking dicloxacillin. She asks you what she needs to do about feeding the baby. You respond, A. "You need to stop breastfeeding and start with a bottle." B. "You can still breastfeed on your left side and express the milk from your right side until the problem is resolved." C. "There is no need to stop breastfeeding, but start with your left side." D. "Stop for 2 weeks, then continue."

C. "There is no need to stop breastfeeding, but start with your left side." When a woman who is breastfeeding develops mastitis, there is no need to stop breastfeeding unless the mastitis is very severe. The woman should begin breastfeeding on the unaffected side because the infant's first sucking is the strongest and would be more painful on the affected side. This also allows the affected breast to "let down." Sulfa drugs should not be prescribed if the nursing infant is younger than 1 month old. Dicloxacillin sodium (Dynapen, Dycill, Pathocil), cephalexin (Keflex), and acetaminophen (Tylenol) may be safely prescribed

A 35-year-old lady presents with severe PID following TOP that was not covered by antibiotics. Her cervical smear report 6 weeks ago shows 'clue cells'. What is the most likely causative organism? A. Staphylococcus Aureus B. Group B Streptococcus C. Bacterial vaginosis D. Chlamydia E. Gonococci

C. Bacterial vaginosis

A 20-year-old woman presents to the sexual health clinic with a grey-white offensive homogeneous vaginal discharge. Microscopy of the discharge revealed clue cells with a positive whiff test. What is the most likely cause of this infection? A. Trichomonas vaginalis (TV) B. Candida albicans C. Bacterial vaginosis (BV) D. Chlamydia trachomatisE. Neisseria gonorrhoeae

C. Bacterial vaginosis (BV)

Which of the following are responsible for symptoms of dysmenorrhea? A. Estrogen B. Human chorionic gonadotropin C. Prostaglandins D. Progestin

C. Prostaglandins

A 40-year-old woman presented to the gynecologist with complaints of profuse vaginal discharge. There was no discharge from the cervix on the speculum examination. The diagnosis of bacterial vaginosis was made based upon all of the following findings on microscopy except A. Abundance of gram variable coccobacilli B. Abundance of anaerobes C. Absence of Lactobacilli D. Abundance of polymorphs E. Present of clue cells

D. Abundance of polymorphs

Combined oral contraception is an effective method of preventing pregnancy. Which of the following conditions not related to pregnancy may warrant the use of birth control pills in a woman? A. Acne B. Ovarian Cancer C. Endometriosis D. All of the above

D. All of the above Combined oral contraception can decrease a woman's risk for pelvic inflammatory disease, decrease menstrual flow, decrease risk of ovarian cancer, decrease risk of ovarian cysts, decrease hirsutism, decrease incidence of endometrial cancer and endometriosis, decrease risk of pelvic inflammatory disease, decrease risk of osteoporosis.

Which of the following is considered a relative contraindication for combined oral contraceptive pills? A. Undiagnosed vaginal bleeding B. Hepatoma of the liver C. Suspected history of transient ischemic attacks (TIAs) D. Depression

D. Depression

Joanne wants to use birth control until she gets married next year. She wants to be able to stop the birth control method after the wedding and wants her fertility restored almost immediately. You recommend: A. Birth control pills B. Vaginal ring C. Depo-provera D. Lea's sheild

D. Lea's sheild Lea's shield is the only device that does not contain hormones. It is similar to the diaphragm and the cap. It is dome shaped and silicone that covers the cervix allowing secretions to exit without sperm entering. Spermicide should be utilized with this form of birth control.

According to the U. S. Medical Eligibility Criteria for Contraception Use, a 32-year-old breastfeeding a 6-month-old infant a category 1 or 2 COC candidate. True False

False

According to the U. S. Medical Eligibility Criteria for Contraception Use, a 45-year-old with fibroids with uterine cavity distortion a candidate for a copper-containing IUD True False

False

Most breast cancer cases are in women with a family history of breast cancer.Select one: True False

False

Nausea with oral contraceptive use can be minimized by taking the pill on an empty stomach. True False

False

According to the U. S. Medical Eligibility Criteria for Contraception Use , a 22-year-old who smokes one pack per day a category 1 or 2 COC candidate. True False

True

According to the U. S. Medical Eligibility Criteria for Contraception Use, a 28-year-old with type 1 diabetes mellitus a category 1 or 2 COC candidate. True False

True

According to the U. S. Medical Eligibility Criteria for Contraception Use, a 33-year-old with low-grade squamous intraepithelial lesions noted on Pap test a candidate for a copper-containing IUD. True False

True

According to the U. S. Medical Eligibility Criteria for Contraception Use, a 45-year-old with tension-type headache a category 1 or 2 COC candidate. True False

True

According to the U. S. Medical Eligibility Criteria for Contraception Use, is a 45-year-old with tension-type headache a category 1 or 2 COC candidate? True False

True

Calcium and vitamin D supplementation is recommended for those taking DMPA (Depo-Provera) injections to minimize the risk of a loss in bone density. True False

True

The use of combined oral contraception (COC) reduces menstrual volume by approximately 60%, thereby reducing the risk of iron deficiency anemia. True False

True

he nurse is assessing a female patient who states that she notices an involuntary loss of urine following a coughing episode. What would be the nurse's best reply? a) "You are experiencing stress incontinence. Do you know how to do Kegel exercises?" b) "You are experiencing reflex incontinence. Have you had a spinal cord injury in the past?" c) "You are experiencing transient incontinence. Have you been administered diuretics or IV fluids lately?" d) "You are experiencing total incontinence. Have you had any surgeries or trauma that may be causing this?"

a) "You are experiencing stress incontinence. Do you know how to do Kegel exercises?"

The nurse prepares to discuss emergency contraception with a client who reportedly had unprotected sex the day before. The nurse knows the following information should be covered when counseling the client. (Select All That Apply) a) A copper intrauterine device (IUD) can be inserted five to seven days after sex. b) The need to take oral contraceptives within 120 hours after intercourse. c) Over the counter availability for persons 16 years and older. d) Oral contraceptives are up to 89% effective in preventing pregnancy.

a) A copper intrauterine device (IUD) can be inserted five to seven days after sex. b) The need to take oral contraceptives within 120 hours after intercourse. d) Oral contraceptives are up to 89% effective in preventing pregnancy.

When assessing a woman for infertility, which of the following testss should be done first? a. Analysis of partner's sperm b. Magnetic resonance imaging (MRI) c. Hysterosalpingogram d. Estrogen level

a. Analysis of partner's sperm

A 20-year-old woman is seen in the clinic because her boyfriend was found to have gonorrhea. Which of the following is the treatment of choice for gonorrhea? a. Ceftriaxone b. Doxycycline c. Acyclovir d. Metronidazole

a. Ceftriaxone

A 22 year old female states that she has multiple sexual partners and inconsistently uses barrier protection. Which form of birth control should the nurse practitioner avoid proscribing in this patient? a. intrauterine device b. progestin-only pill c. diaphragm d. oral contraceptives

a. intrauterine device

Athletic amennorrhea increases the risk of: a. osteoporosis b. an eating disorder c. covert hypothyroidism d. breast cancer

a. osteoporosis

A 45-year-old woman is seen in the clinic with complaints of a vaginaldischarge. The clinician identifies clue cells on the vaginal smear. Which of the followingdiagnoses is associated with this finding? a. Trichomonas b. Bacterial vaginosis c. HPV d. Herpes simplex virus

b. Bacterial vaginosis

A 25-year-old woman is seen in the clinic complaining of painful menstruation. Which of the following pelvic pathologies is the most common cause of dysmenorrhea? a. Pelvic inflammatory disease b. Endometriosis c. Sexually transmitted infections d. Ovarian cyst

b. Endometriosis

An initial pharmacological approach to a patient who is diagnosed with primary dysmenorrhea could be: a. acetaminophen b. NSAIDs at the time symptoms begin or onset of menses c. NSAIDs prior to the onset of menses d. combination acetaminophen and NSAIDs

b. NSAIDs at the time symptoms begin or onset of menses

A 22-year-old woman is diagnosed with premenstrual syndrome. Which of the following lifestyle changes should the clinician suggest to help minimize the patient's symptoms? a. At least 4 cups of green tea daily b. Regular exercise c. Take vitamin A supplements d. Eat a diet high in iron

b. Regular exercise

When collecting cervical cells for a PAP smear, when are the endocervical cells typically collected? a. after the ectocervical specimen with a broom b. after the ectocervical specimen with a brush c. before the ectocervical specimen with a broom d. before the ectocervical specimen with a brush

b. after the ectocervical specimen with a brush

Clue cells are found in patients who have: a. leukemia b. bacterial vaginosis c. epidermal fungal infections d. pneumonia

b. bacterial vaginosis

The clinical syndrome resulting from replacement of normal vaginal flora with anaerobic bacteria is: a. candidal vaginitis b. bacterial vaginosis c. pelvic inflammatory disease d. herpes simplex

b. bacterial vaginosis

A 22 year old female has been diagnosed with polycystic ovarian syndrome (PCOS). What is a common finding? a. low testosterone level b. elevated insulin levels c. positive pregnancy test d. elevated blood pressure

b. elevated insulin levels

A patient asks the NP's advice about an herb to help her with hot flashes. The NP knows these: a. are safe to use in all patients b. may be contraindicated in patients with a history of breast cancer c. substances have a mild estrogenic effect that will halt hot flashes d. help prevent osteoporosis

b. may be contraindicated in patients with a history of breast cancer

Which example best supports the diagnosis of Sexual Dysfunction: Dyspareunia? a) A 50-year-old woman with a history of stroke is afraid to have sex with her partner for fear it will elevate her blood pressure. b) A 39-year-old alcoholic woman is no longer interested in having sex with her partner. c) A 50-year-old woman in the process of menopause has pain and burning during intercourse. d) A patient with a colostomy believes she cannot have a sexual relationship with her husband because he will be repulsed by her stoma.

c) A 50-year-old woman in the process of menopause has pain and burning during intercourse. Dyspareunia refers to pain and burning during intercourse. This is a common cause of sexual dysfunction, especially during menopause. A colostomy, fear of blood pressure elevation, and lack of interest in sex may lead to the nursing diagnosis of Sexual Dysfunction, but not related to dyspareunia

A teenage female is in the community health clinic seeking contraception. The community health nurse demonstrates effective health promotion and prevention with the client when including which of the following in the health education teaching plan? a) Correct use of the contraception method selected. b) The importance of using a barrier method to prevent risk of HIV transmission. c) Contraception plans prior to initiation of sexual activity. d) The need for a back-up contraceptive method when using antibiotics.

c) Contraception plans prior to initiation of sexual activity. The prevention of unwanted pregnancy must be a conscious decision. Anyone who is unprepared for pregnancy should refrain from intercourse or obtain a contraceptive method from a healthcare provider or from the pharmacy; it is too late to think about contraception during sexual intercourse. To practice responsible sexuality, the contraceptive method must be used consistently and according to instructions

A 24-year-old woman presents to the clinic with dysuria, dyspareunia, anda mucopurulent vaginal discharge. Her boyfriend was recently treated for nongonococcal urethritis. What sexually transmitted disease has she most probably been exposed to? a. Gonorrhea b. HPV c. Chlamydia d. Trichomonas

c. Chlamydia

A 32 year old female presents to your clinic for her annual GYN exam. A 1cm soft, non-tender round mass is found at the 4 o'clock position at the posterior vaginal introitus. What is the most likely diagnosis a. nabothian cyst b. mesonephric cyst c. bartholin duct cyst d. bartholin gland abscess

c. bartholin duct cyst

A 4 year old female is brought in to the clinic by her mother, who reports that she is constantly scratching "her private part" The patient states that it itches. On exam, the vagina is red and irritated. How should the NP proceed? a. call child protection for suspected sexual abuse b. prescribe a cortisone cream c. collect a vaginal swab of the external vagina for microscopic evaluation d. prescribe a topical antifungal

c. collect a vaginal swab of the external vagina for microscopic evaluation

Three of the following interventions are appropriately used to prevent osteoporosis after menopause. Which one is not? a. avoidance of corticosteroids b. performance of weight bearing activities for 40 min at least 5 days/week c. estrogen replacement therapy d. adequate calcium & vitamin D intake

c. estrogen replacement therapy

A nurse practitioner identifies the following image during a microscopic exam of vaginal discharge. These are probably: a. bacteria b. sperm c. hyphae d. candida albicans

c. hyphae

An 84 year old female patient is a resident in an assisted living facility . She has early dementia. She walks daily and has had urinary incontinence for years. She has loss of urine with coughing, sneezing and if she is unable to get to the bathroom quickly enough. Her urinary incontinence is likely to be: a. urge b. stress c. mixed d. overflow

c. mixed

A female should be told to take her oral contraceptive pill (ocp) at bedtime if she experiences: a. weight gain b. headaches c. nausea d. spotting

c. nausea

A 28 year old female presents with a slightly tender 1.5 cm lump in her right breast. She noticed it 2 days ago. She has no associated lymphadenopathy and there is no nipple discharge. How should she be managed? a. mammogram b. ultrasound and mammogram c. re-examination after her next menses d. clinical exam only

c. re-examination after her next menses

Women who use diaphragms for contraception have an increased incidence of: a. sexually transmitted diseases b. pregnancy c. urinary tract infection d. pelvic inflammatory disease

c. urinary tract infection

To help prevent osteoporosis, what should a nurse advise a young woman to do? a) Avoid trauma to the affected bone. b) Encourage the use of a firm mattress. c) Keep the serum uric acid level within the normal range. d) Consume at least 1,000 mg of calcium daily.

d) Consume at least 1,000 mg of calcium daily.

The frequency for cervical screening depends on the patient and her age. what is the longest recommended time interval between cervical screens for a patient who is 30-64 years old. a. 1 year b. 2 years c. 3 years d. 5 years

d. 5 years

A female patient is 35 years old. She has never had an abnormal PAP smear and has had regular screening since age 18. If she has a normal PAP smear with HPV testing today, when should she have the next cervical cancer screening? a. One year b. 2-3 years c. 3 years d. 5 years

d. 5 years American College of Obstetricians and Gynecologists recommends screening for women over age 30 years no more frequently than every 5 years if both test are negative and adequate screening has taken place. If she had been screened with PAP only, the recommendation screening would be in 3 years. US Preventive Services Task Force recommends screening this patient in 5 years. Human papilloma virus (HPV) testing in combination with cervical cytology has been shown to be more sensitive than cytology alone. However, HPV alone is not recommended as a lone screening modality. Finally, HPV testing is more specific in women over age 30 years. Hollier, A. (2013) Adult-gero and family nurse practitioner certification practice questions p. 77

A 45-year-old woman is seen in the clinic with abnormal uterine bleeding and pain during intercourse. The clinician should consider which of the following diagnoses? a. Leiomyoma b. Pregnancy c. Ovarian cancer d. All of the above

d. All of the above

A 26-year-old woman tells the clinician that she has endometriosis, because she has frequent pelvic pain. The clinician also should consider which of these differential diagnoses? a. Diverticulitis b. Cholelithiasis c. Kidney stones d. Ovarian cysts

d. Ovarian cysts

Alice has been diagnosed with breast cancer. Its TNM staging is T2, N1, M0. She would like to know what this means. Which statement made by the clinician is accurate? A. "Your tumor is 4 cm, has metastasized to a moveable lymph node, but has not metastasized to another location." B. "Your tumor is 5.5 cm and has not metastasized to lymph nodes or anywhere else." C. "Your tumor is 1.5 cm, has metastasized to a moveable lymph node, but has not metastasized to another location." D. "Your tumor is 3 cm, has metastasized to a moveable lymph node, and has only metastasized to supraclavicular lymph nodes."

A. "Your tumor is 4 cm, has metastasized to a moveable lymph node, but has not metastasized to another location."

Mrs. Peterson presents to your office today to be fitted for a diaphragm. She has been on numerous hormones in the past and does not like the side effects. When properly fitting a patient for a diaphragm you know: A. Allow a fingertip between it and the pubic arch B. Be small enough to allow for vaginal expansion C. Lie snuggly over the pubic arch and under the cervix. D. Provide firm tension against vaginal walls

A. Allow a fingertip between it and the pubic arch Diaphragm is a barrier method that may provide six hours of contraception. It is a dome shaped rubber cup with a rim that is flexible. It is inserted into the vagina before intercourse so the posterior rim rests on the posterior portion of the vagina and the anterior rim fits snugly behind the pubic bone. The diaphragm covers the cervix and is used in conjunction with spermacide. When inserted correctly there is just enough space to insert one fingertip between pubic arch and the anterior edge of the diaphragm rim.

A 26-year-old female presents to the office with complaints of malodorous, grayish discharge for the past 3 days. She denies dysuria, dyspareunia, or pelvic discomfort. She is sexually active with two partners and intermittently uses condoms. She is concerned as one of her partners was recently treated for Chlamydia. A speculum examination is performed and a sample of vaginal fluid is examined underneath the microscope. Many vaginal epithelial cells with adherent clusters of bacteria are seen on the field. What is the most likely diagnosis? A. Bacterial Vaginosis B. Chlamydia trachomatis C. N. Gonorrhea D. Trichomonas E. Listeria monocytogenes

A. Bacterial Vaginosis

A 20-year-old woman who is 8 weeks pregnant presents with a 2 week history of fishy smelling PV discharge. It is non-itchy and creamy coloured. She has had previous episodes outside of pregnancy which resolved spontaneously. She is in a monogamous relationship with the same partner for 5 years. On speculum examination there is a small amount of smooth grey discharge seen coating the vaginal walls. What is the most likely diagnosis? A. Bacterial vaginosis B. Candida albicans C. Trichomonas vaginalis D. Chlamydia trachomatis E. Neisseria gonorrhea

A. Bacterial vaginosis

A 28-year-old woman presents with a thin, grayish-white vaginal discharge with a fishy odor, accompanied by mild vaginal pruritus. She denies postcoital or intermenstrual vaginal bleeding. She is sexually active with her boyfriend (only) for the past year, and her sexual desire is normal. She feels safe at home and denies any conflicts with her partner. She also denies dysuria and urinary frequency. Her last period 2 weeks ago. On examination reveals a pink cervix and a thin white discharge coating vaginal walls and vestibule. The pH of the discharge is 6.0. No history of abnormal Pap smears; most recent was 6 months ago. She uses patch for contraception. What is the most likely diagnosis? A. Bacterial vaginosis B. Candidal vaginitis C. Trichomonal vaginitis D. Chlamydial cervicitis E. Gonorrheal cervicitis

A. Bacterial vaginosis

A 43-year-old woman attends the GP with a 3-month history of a grey-white vaginal discharge which she says has a "fishy" odour. She is systemically well and has no menstrual abnormalities. What is the most likely diagnosis? A. Bacterial vaginosis (BV) B. Candida C. Chlamydia D. Gonorrhoea E. Syphilis

A. Bacterial vaginosis (BV)

Which of the following drugs may have there effects enhanced when used while taking oral contraceptives? A. Beta Blockers B. Oral anticoagulants C. Antacids D. Anticonvulsants

A. Beta Blockers A. Beta blockers, antidepressants, benzodiazepine, corticosteroids, theophyline, and valium may be enhanced when taken in conjunction with combined oral contraceptives.

Your patient has been successfully using Depo Provera as birth control for the past six months. She was given an appointment 12 weeks after her last injection but forgot she had it. It is now 3 days past her scheduled appointment and she is requesting her injection. What should your action be? A. Come to the clinic today for another injection B. Wait until her next menstrual cycle starts to restart the Depo C. Use foam and condoms until a normal menses occurs, then come to the clinic for another injection. D. Counsel the patient on another birth control method since she is unreliable

A. Come to the clinic today for another injection Depo Provera should be given every twelve weeks but can be given up to the thirteen week and still be considered a viable form of birth control.

A 36 year old female being evaluated for birth control choices. She is a smoker. Has a history of chlamydia cervicitis. She has two small children. The best contraceptive choice for her would be: A. Depo-Provera injections B. Oral contraceptives C. IUD D. Estrogen patches

A. Depo-Provera injections Because the patient is a smoker she should not be on oral contraception or any other form of estrogen. Because she has had a history significant for chlamydia she should not have an IUD inserted because the number one risk with utilizing IUD is infection.

A good choice for long acting reversible contraception for a 30 year old who already has two children could be any of the following except: A. Diaphragm B. Implanon C. Intrauterine device D. Depo-vera

A. Diaphragm

Which of the following is an alternative treatment for breast tenderness with premenstrual syndrome? A. Evening primrose oil 250 mg orally up to 3 times daily 2 to 3 days before menses B. Black cohosh 40 to 200 mg orally daily C. Vitamin B complex 50 mg orally daily D. Jasmine essential oil aromatherapy

A. Evening primrose oil 250 mg orally up to 3 times daily 2 to 3 days before menses

If a woman is using basal body temperature as birth control when would you tell her to avoid unprotected sex? A. From the beginning of her cycle until the basal body temp. has been elevated for three days. B. Whenever the Basal body temp. is elevated C. Whenever the basal body temp is lowered D. From the end of the menstrual cycle until the basal body temp has been low for five days.

A. From the beginning of her cycle until the basal body temp. has been elevated for three days. Sex unprotected should be avoided from the beginning of the menstrual cycle until the basal body temp has elevated for three days. Temps are suppressed by estrogen whereas post ovulatory temps increase under the influence of progesterone. Temps usually rise within a day or two after ovulation has occurred and remain elevated until menstration has begun.

A 23-year-old sexually active woman presents for her first Pap smear. Her history includes nulligravida, age at first intercourse 14, and more than 10 sexual partners. Which of the following conditions should the clinician be particularly alert for during her examination? A. Human papillomavirus B. Endometrial hyperplasia C. Vaginismus D. Polycystic ovarian syndrome

A. Human papillomavirus

A twenty seven year old presents for insertion of mirena. She reports that her menses started 3 days ago and is normal. How soon after insertion will be be able to rely on it for protection? A. Immediately B. After forty eight hours C. After one week D. In one month

A. Immediately Mirena gives five years protection. It can be inserted anytime during the menstrual cycle but if inserted within the first seven days of menstration it is effective immediately. If inserted any other time during the month another form of birth control should be used for seven consecutive days after insertion

Sydney is a 21 year old patient who is taking oral contraception. She comes to your office today complaining of acne. How should you adjust the estrogen component of the birth control pill? A. Increase the estrogen content B. Decrease the estrogen content C. Delete the estrogen content D. No adjustment should be made in the estrogen content

A. Increase the estrogen content Patients suffering from acne should have an increase in the estrogen component of the birth control pill or a decrease in the progesterone component. Remember progesterone is androgenic.

Which one of the following statements is not true with respect to bacterial vaginosis (BV)? A. Isolation of Gardnerella vaginalis on high vaginal swab culture is diagnostic of BV B. Vaginal pH is increased and is typically more than 4.5 C. Presence of 'clue cells' on microscopy is diagnostic of bacterial vaginosis D. It is associated with preterm labour or miscarriage E. It is the commonest cause of abnormal vaginal discharge in women of reproductive age

A. Isolation of Gardnerella vaginalis on high vaginal swab culture is diagnostic of BV BV is the commonest cause of abnormal discharge in the childbearing age group. It is not a sexually transmitted disease. It is caused by an overgrowth of mixed anaerobes replacing the dominant vaginal lactobacillus species, resulting in an increased vaginal pH to 4.5 to 7.0. Isolation of Gardnerella vaginalis on HVS culture is not diagnostic and is found in up to 30-40% of 'normal' women. BV usually presents with a profuse, homogenous, whitish grey, offensive smelling vaginal discharge. There is a characteristic fishy smell on adding 10% potassium hydroxide to the discharge and the presence of 'clue cells' on wet mount of the discharge is diagnostic of bacterial vaginosis. In pregnancy, BV is associated with late miscarriage, pre-term rupture of membranes, pre-term delivery and postpartum endometritis.

Which of the following is true about the histopathology of Gardnerella vaginalis? A. It grows colonies on both chocolate and human blood Tween agar B. It is spore forming C. It is motile D. It is an encapsulated budding yeast E. It has a thick gram-positive cell wall

A. It grows colonies on both chocolate and human blood Tween agar

Which of the following is true about the bacterium Gardnerella vaginalis? A. It produces a pore-forming toxin, vaginolysin B. It is gram negative C. It is considered to be a contagious bacteria causing sexually transmitted diseases D. It cannot adhere to human epithelium E. It is obligate anaerobic Coccobacillus

A. It produces a pore-forming toxin, vaginolysin

All of the following are correct in regards to Implanon except: A. Lasts for five years B. Is a better choice for women with multiple sex partners then an IUD because there is less of a chance of pelvic inflammatory disease C. It is a safe form of birth control for lactating women D. Works by supressing ovulation, thinning the endometrial lining, and thickening the cervical mucus

A. Lasts for five years Implanon is a radio opaque that will last for three years if not removed. If not removed after three years it will no longer be active against pregnancy. It is safe during lactation because it is a progesterone only form of contraception.

A 29-year-old woman presents with new vaginal discharge that started 1 week ago. She describes the discharge as thin and off-white in color, with an unpleasant odor. She denies fevers, chills, pain with urination, vaginal itching, pain during sex, or other systemic symptoms. She is sexually active with both men and women and recalls 4 partners in the past 6 months. After evaluation in the clinic, she is diagnosed with bacterial vaginosis. Which one of the following options is the most appropriate therapy for this woman with bacterial vaginosis? A. Metronidazole 500 mg orally twice a day for 7 days B. Metronidazole 2 g orally as a single dose C. Azithromycin 1 g orally as a single dose D. Ofloxacin 400 mg orally twice a day for 7 days E. Clindamycin 900 mg orally as a single dose

A. Metronidazole 500 mg orally twice a day for 7 days

Tina is an 18-year-old female who would like to start using the transdermal contraceptive patch. Which of the following instructions should the clinician discuss with Tina? A. Obesity can decrease the effectiveness of the patch. B. The patch should be applied to the buttocks or breasts. C. The patch is changed every 10 days. D. If a patch becomes detached for less than 24 hours, it cannot be reapplied.

A. Obesity can decrease the effectiveness of the patch.

A young female has been diagnosed with bacterial vaginosis in the emergency department. The client has been prescribed a medication and is ready for discharge. What instructions will the registered nurse reinforce with the client during the discharge process? A. Take antibiotics as prescribed B. If taken with alcohol, there is no disulfiram reaction with metronidazole intravaginal gel C. Use condoms while taking metronidazole intravaginal gel D. Routine vaginal douching E. Take a bubble bath every other day

A. Take antibiotics as prescribed

Cervical mucus changes throughout the menstrual cycle. The appearance of the cervical mucus consistent with fertile-type mucus is best described as A. copious, clear, and stretchable. B. scant, thick, and sticky. C. copious, thick, and clotted. D. scant, clear, and elastic.

A. copious, clear, and stretchable. Right before ovulation, as estrogen levels increase, the cervical mucus is copious, thin, and stretchable. It feels like a lubricant and can be stretched between the fingers. This is called spinnbarkheit. This is the time of maximum fertility. During most of the menstrual cycle, cervical mucus is scant in quantity and thick.

Which effect is seen in every women using Depo Provera for more than four years? A. Melasma B. Amenorrhea C. Weight loss D. Headaches

B. Amenorrhea

Which of the following is not an absolute contraindication for the utilization of birth control pills: A. Smoking under 35 with a history of a DVT B. Family history of migraine with aura C. Depression and history of gallbladder disease in pregnancy D. Controlled hypertension with diabetes mellitus with vascular changes

B. Family history of migraine with aura Family history of migraine is not the same thing as a personal history of migraine with aura.

A female patient is being prescribed a medication for bacterial vaginosis (BV) and would like to know about potential complications likely associated with this infection. Which of the following is not a complication associated with bacterial vaginosis? A. Acquiring HIV, Chlamydia and Gonorrhea B. Female to male sex transmission C. Pelvic inflammatory disease D. Post-termination endometritis E. Vaginal cuff cellulitis after vaginal hysterectomy

B. Female to male sex transmission

A 33-year-old female presents with a 10-day history of malodorous vaginal discharge. She has had three male sex partners during the past 6 months and uses a diaphragm, spermicide, and condoms for birth control. She reports recent use of a vaginal douche three times a week for her personal hygiene. About 3 weeks ago, she was diagnosed with "strep" throat and received a course of penicillin. Currently she denies fever, chills, urinary symptoms, or other systemic symptoms. On pelvic examination, a thin, white vaginal discharge is visible with associated fishy odor after addition of 10% potassium hydroxide (KOH). Microscopy of a sample of the vaginal discharge is significant for the presence of abundant clue cells. Which one of the following is a risk factor for bacterial vaginosis? A. Recent use of diaphragm and spermicide B. Frequent douching C. Recent intercourse with a male partner with use of latex condoms D. Recent antibiotic course with penicillin or amoxicillin E. Monogamous relationship

B. Frequent douching

A 32-year-old female presents with a 4-day history of malodorous vaginal discharge. She has engaged in sexual intercourse with multiple male partners over the past 12 months. She is otherwise healthy and denies other medical problems or use of medications. She denies having fevers, chills, pain with urination, pelvic pain, pain during sex, or vaginal itching. Based on the history, you suspect bacterial vaginosis. What bacteria are most commonly associated with bacterial vaginosis? A. Group B streptococcus, Mycoplasma genitalium, and Lactobacillus crispatus B. Gardnerella vaginalis, Mycoplasma hominis, and anaerobes C. Chlamydia trachomatis, Escherichia coli, and Klebsiella oxytoca D. Group B streptococcus, Corynebacterium striatum, and Escherichia coli E. Actinomyces israelii, Mycobacterium tuberculosis and Haemophilus haemolyticus

B. Gardnerella vaginalis, Mycoplasma hominis, and anaerobes

You have recently started you patient on oral contraceptives. She followed your instructions and started her first pill on the Sunday after she started her period. She is in your office today for her three month follow-up visit and is complaining of breakthrough bleeding during the first two weeks of her pill cycle. Your next course of action would be: A. Reassure the patient that this is normal and she should continue taking her pill at the same time everyday for the first 21 days of every month. B. Increase the estrogen component of her pill. C. Increase the progesterone component of her pill. D. Lower the progesterone component of the pill. E. Decrease the estrogen component of her pill.

B. Increase the estrogen component of her pill. Breakthrough bleeding from day 1 to 14 of a woman's menstrual cycle indicates that the estrogen component is not high enough. Bleeding irregularities should go away within the first three months of utilizing oral contraception.

Which of the following hormones induces ovulation: A. FSH B. LH C. Estrodiol D. Progesterone

B. LH

A 32-year-old G2P1 at 32 weeks gestation presents with a 4-day history of vaginal discharge. She denies itching, burning, or pain. On physical exam, a homogenous white discharge is noted to coat the vaginal side walls. A wet mount of the discharge shows clue cells, and a fishy odor is noted when KOH is added to the discharge. What is the best treatment for this patient? A. Tinidazole 2 g orally once B. Metronidazole 400 mg orally twice daily for 7 days C. Ampicillin 500 mg orally twice daily for 7 days D. Triple-sulfa intravaginal cream twice daily for 7 days E. Clindamycin (2%) intravaginal cream once daily for 7 days

B. Metronidazole 400 mg orally twice daily for 7 days

A female who is pregnant and has developed bacterial vaginosis should be treated with which of the following? A. Oral amoxicillin B. Oral clindamycin C. Clotrimazole pessary D. Intravaginal clindamycin E. Daily douching

B. Oral clindamycin

A 23-year-old woman complains of vaginal discharge and a high vaginal swab shows bacterial vaginosis. Which of the following is consistent with bacterial vaginosis? A. Inflammation of the posterior vaginal wall on colposcopic examination B. Presence of clue cells on microscopy C. Presence of motile parasites in the vaginal secretions D. Presence of thick, white vaginal discharge E. Vaginal discharge pH less than 4.0

B. Presence of clue cells on microscopy

A 25-year-old sexually active woman complains of a "fishy" smelling gray-white vaginal discharge. You examine this on wet mount and see epithelial cells with clusters of bacteria obscuring their borders. The vaginal pH is 5.5. This infection has been most closely implicated in which of the following complications of pregnancy? A. Intrauterine growth restriction B. Preterm birth C. Congenital cataracts D. Learning disabilities during childhood E. Preeclampsia

B. Preterm birth

Mrs. Henderson inquires about why she needs progesterone added to her estrogen hormone replacement therapy. You explain that women with intact uterus's need to add progesterone to prescribed estrogen because: A. It assists with stopping hot flashes B. Reduces the incidence of endometrial hyperplasia and cancer C. Decreases the risk of osteoporosis D.. Controls mood swings

B. Reduces the incidence of endometrial hyperplasia and cancer

A woman eight weeks post partum comes to your office asking about birth control. She is breastfeeding and wants a reliable birth control method that will not interfere with lactation. You recommend: A. Oral contraceptives B. The Mini pill C. The transdermal contraceptive patch D. The Nuvaring

B. The Mini pill The mini pill is the only option that is progesterone only. All other choices contain estrogen which cannot be given during lactation. Must be given on time everyday and can be given in the absence of menstration. It is 100% effective to prevent pregnancy in lactating women if utilized properly and can be immediately reversed by the patient at anytime making it a better option then Depo-Provera, IUD, or Implanon.

Betsy is breastfeeding and complains of tenderness of the nipples all the time. You recommend that she A. apply ice to the nipples between feedings. B. apply dry heat to the nipples between feedings. C. use nipple shields. D. stop breastfeeding.

B. apply dry heat to the nipples between feedings. If a woman is breastfeeding and complains of tenderness of the nipples all the time, suggest that she apply dry heat to the nipples between feedings. A common symptom during the first days of breastfeeding, tenderness of the nipples usually begins when the baby starts to suck and then subsides as soon as the milk begins to flow. If maternal tissues are unusually tender, dry heat usually helps. Ice tends to increase nipple tenderness. Nipple shields should be used only as a last resort because they interfere with normal sucking. If it is necessary to wear nipple shields, suggest glass or plastic shields with rubber nursing nipples rather than all-rubber shields. This is a temporary problem that can be resolved. Stopping breastfeeding for a temporary problem is an irreversible solution.

Gloria just delivered her baby and wants to start a vigorous exercise program to get back in shape. You advise her that she can start strenuous exercises A. in 1 week. B. in 3 weeks. C. in 6 weeks. D. whenever she wants

B. in 3 weeks. After delivery, exercises to strengthen the muscles of the back, pelvic floor, and abdomen are advocated, but strenuous exercises should be postponed until about 3 weeks after delivery. This allows the abdominal muscles to partially regain their original length and tone and prevents undue client fatigue.

A 24-year-old nulliparous woman is noted to have a bothersome vaginal discharge. On examination, she is found to have a homogenous discharge with a fishy odor. Which of the following characteristics is likely to be noted on examination of the discharge? A. Motile protozoa on wet mount B. pH more than 4.5 C. Strawberry cervix on speculum examination D. Abundant lactobacilli on saline prepared microscopy E. Budding hyphae on KOH examination

B. pH more than 4.5

The clinician is seeing Jalissa, a 17-year-old patient, for a well woman's exam. She mentions that she has been depressed and has been yo-yo dieting because she feels "so fat." She marked "no" to whether she feels safe in her home on her intake form, and mentions she is worried about getting STIs from her boyfriend of three months. Which of the following is the correct way to interact with the patient, based on her history? A. "The amount of time allotted for your well woman exam does not allow time to talk about depression today." B. "I know you said you are concerned about your body image, but your weight looks good to me." C. "You mentioned on your intake form that you do not feel safe in your home. Why is that?" D. "You've been dating your boyfriend for three months, but you haven't had an STI screening yet? That's not safe."

C. "You mentioned on your intake form that you do not feel safe in your home. Why is that?"

What proportion of women experience no symptoms of bacterial vaginosis (BV)? A. 15 percent B. 25 percent C. 50 percent D. 85 percent E. 90 percent

C. 50 percent

A 17-year old female who is pregnant presents to the clinic with grayish fishy odour vaginal discharge. Work up reveals that she has developed bacterial vaginosis. She is treated and discharged. Based on recent guidelines, when should she be rescreened? A. She do not need any further tests of cure B. After one week C. After one month D. After three months E. Only if she indulges in unprotected sexual intercourse

C. After one month It is not necessary to have a further test to prove resolution if symptoms resolve (unless treatment is prescribed in pregnancy to reduce the risk of preterm birth, in which case a repeat test should be made after one month and further treatment offered if the bacterial vaginosis has recurred

Which of the following is the diagnostic standard for gestational diabetes? A. The presence of glycosuria on two urine samples within a 24-hour interval B. A 50-g oral glucose challenge C. An abnormal 3-hour glucose tolerance test (GTT) after a 100-g glucose load D. A postprandial glucose blood serum level greater than 140 mg/dL

C. An abnormal 3-hour glucose tolerance test (GTT) after a 100-g glucose load The diagnostic standard for the diagnosis of gestational diabetes is an abnormal 3-hour GTT. A 50-g oral glucose challenge test is a screening test to identify women who are candidates for the 3-hour GTT. A postprandial glucose level greater than 140 mg/dL is not indicative of gestational diabetes. Because of changes in renal filtration rates during pregnancy, many women have evidence of glucose in voided urine.

A patient with pelvic inflammatory disease (PID) presents with offensive, fishy smelling par vagina discharge. High vaginal swab confirms: A. Chlamydia B. Gonorrhoea C. Anaerobes including Gardnerella and Mycoplasma D. CandidaE. Trichomonas vaginosis

C. Anaerobes including Gardnerella and Mycoplasma

Which of the following drugs decreases the effectiveness of oral contraceptives? A. Beta Blockers B. oral anticoagulants C. Antibiotics D. Oral hypoglycemic agents

C. Antibiotics Antibiotics, antacids, anticonvulsants, and barbituates can decrease the effectiveness of oral contraceptives.

A 35-year-old woman presents to her GP with a 2-week history of a fishy odorous vaginal discharge, which occurs especially after sexual intercourse. Microscopy of the discharge reveals clue cells. What is the most likely causative organism of her discharge? A. Treponema pallidum B. Klebsiella granulomatis C. Bacterial vaginosis D. Haemophilus ducreyi E. Herpes simplex virus 2

C. Bacterial vaginosis

A woman who is 14 weeks pregnant presents with a thin vaginal discharge and fishy odour. She gives history of this getting worse before her periods and with sexual intercourse. On examination, the vulva and vagina looked normal and not inflamed. The diagnosis in her case is which of the following? A. Tricomoniasis B. Candidiasis C. Bacterial vaginosis D. Chlamydia E. Donovanosis

C. Bacterial vaginosis

Which of the following is the most common type of vaginal infection? A. Gonorrhea B. Trichomoniasis C. Bacterial vaginosis D. Candidiasis E. Chlamydia

C. Bacterial vaginosis Bacterial vaginosis is the most common vaginal infection found in women of reproductive age and is estimated to occur in anywhere from 5% to 70% of women. It may be that as many as about 1 in 3 women have BV at some time in their lives. Interestingly, worldwide this condition is most common in parts of Africa and is found to be least common in Asia and Europe.

A 24-year-old woman presents to the clinic with dysuria, dyspareunia, and a mucopurulent vaginal discharge. Her boyfriend was recently treated for nongonococcal urethritis. What sexually transmitted disease has she most probably been exposed to? A. Gonorrhea B. HPV C. Chlamydia D. Trichomoniasis

C. Chlamydia

Kim states that she has heard many old wives' tales of harmful things during pregnancy. Which of the following is harmful late in pregnancy? A. Intercourse B. Swimming C. Douching D. Dental visits

C. Douching Douching, which is seldom necessary, may be harmful during pregnancy. Intercourse late in the pregnancy may initiate labor, possibly because an orgasm might cause a uterine contraction reflex. Intercourse is usually cautioned against only in women who have had a previous premature delivery or are currently experiencing uterine bleeding. Water does not enter the vagina; therefore swimming is not CHAPTER 4: CARE OF THE EMERGING FAMILY 67 1819_Ch04_055-082.qxd 12/22/08 5:20 PM Page 67 contraindicated. However, diving should be avoided because of the possibility of trauma. Good dental care is important during pregnancy; however, the dentist should be told that the woman is pregnant.

All of the following are absolute contraindications of the birth control pill except: A. Benign hematoma B. History of embolism C. Family history of migraine with aura D. History of gallbladder disease during pregnancy

C. Family history of migraine with aura

A 25-year-old woman presents with yellow-greenish musty odorous vaginal discharge; On visualization of the discharge under the microscope demonstrates multiple vaginal squamous epithelial cells covered by bacteria cells. What is the most likely causative organism? A. Neisseria gonorrhea B. Chlamydia trachomatis C. Gardnerella vaginalis D. Escherichia coli E. Treponema pallidum

C. Gardnerella vaginalis

A 32-year-old sexually active female complains of vaginal itching and burning with malodorous discharge. On a wet mount you find epithelial cells coated with bacilli in cytoplasm. What is the most likely causative organism? A. Neisseria gonorrhea B. Chlamydia trachomatis C. Gardnerella vaginalis D. Escherichia coli E. Treponema pallidum

C. Gardnerella vaginalis

A 37-year-old woman is seen in the gynaecology outpatient clinic complaining of a profuse, fishy-smelling, thin grey vaginal discharge; microscopy shows the presence of clue cells; the whiff test is positive. Which is the most likely causative agent? A. Candida albicans B. Chlamydia trachomatis C. Gardnerella vaginalis D. Escherichia coli E. Trichomonas vaginalis

C. Gardnerella vaginalis

Lynne, comes into your office in tears. She reports to you that she had sex last night unprotected and forgot to take her birth control pill. She wants to know about the morning after pill. You tell her: A. If your period does not start at the regular time come back and see me. B. I will go ahead and order you the estrogen only postcoital contraceptive pill. C. If you utilize over the counter levonorgestrel 1.5 mg within 72 hours of unprotected sex it will reduce your chance of pregnancy by 75%. D. I will refer you to a gynecologist

C. If you utilize over the counter levonorgestrel 1.5 mg within 72 hours of unprotected sex it will reduce your chance of pregnancy by 75%. Emergency contraception should be taken within 72 hours of having unprotected sex. There are 3 regimens available in the U.S.A. (1) Yuzpe regimen - two pills within 72 hours and two 12 hours later is 75% effective (2) Progesterone only levonorgestrel 1.5 mg should be used within 72 hours of unprotected sex - can be purchased over the counter (3) Insertion of copper IUD up to seven days after unprotected sex.Plan B: is 1.5 mg levonorgestrel that will delay or prevent ovulation or interfere with the fertilization of the egg. Plan B One-Step can be purchased over the counter at drugstores without a prescription or proof of age. Because it is most effective when taken as soon as possible (up to 72 hours after sex), consider having a ready supply in your medicine cabinet. If you take it within 72 hours after you've had unprotected sex, Plan B One-Step can reduce the risk of pregnancy by up to 89%. If you take Plan B One-Step within 24 hours, it is about 95% effective.But you should know that Plan B One-Step is not as effective as regular contraception. So don't take it as your main form of birth control. And, it does not protect you against sexually transmitted diseases. Think of it as a backup -- not for routine use. That's why it's called Plan B.

One of the major side effects from this form of contraception is abnormal uterine bleed. A. Oral contraceptive pills B. NuvaRing C. Implanon D. Essure

C. Implanon Is a progesterone only product. Irregular bleeding is the most common side effect especially in the first six to twelve months. For most women periods become fewer and lighter. After one year 1 out of three women who us Implanon will stop having periods completely. Some women may experience heavier, longer periods and spotting b/w periods.

A 23-year-old G2P1 at 28 weeks GA comes to the office for an urgent visit. She complains of vaginal irritation and increased thin, grey discharge. She denies leakage of fluid, vaginal bleeding, or contractions. The baby is moving well. She is in a monogamous relationship and last had intercourse 2 weeks ago. She has not noticed any dysuria or urinary frequency. She has no history of sexually transmitted infections. You perform a sterile speculum examination, and there is no evidence of ruptured membranes. You collect a sample of discharge and perform a wet mount and KOH, which shows clue cells, a positive whiff test, and no hyphae. The pH of the discharge is less than 5.0. You diagnose her with bacterial vaginosis. If this had gone untreated, how would it have increased her risk during this pregnancy? A. Increased risk of neonatal blindness B. Increased risk of neonatal sepsis and admission to the NICU C. Increased risk of preterm premature rupture of membranes (PPROM) D. Increased risk of placental abruption E. Increased risk of congenital malformations

C. Increased risk of preterm premature rupture of membranes (PPROM)

A mechanism of action for progesterone only contraceptives includes thickening of the cervical mucus, production of a thin endometrium, and: A. Premature degeneration of the Corpus Luteum B. Inhibits FSH production C. Inhibits the LH surge D. Mechanically inhibits implantation

C. Inhibits the LH surge

Women who utilize oral contraceptives are less likely to experience: A. Human papilloma virus B. Migraine headaches C. Iron deficiency anemia D. Herpes simplex virus

C. Iron deficiency anemia Oral contraceptives cause less menstrual bleeding.

A 17 year old high school student is considering Seasonale as her method of birth control. Which of the following statements is false regarding Seasonale: A. Taking seasonale will lesson your menses to four yearly B. A menstrual period will occur within seven days of stopping Seasonale C. Is a progesterone only pill D. Must be taken for 84 consecutive days followed by seven days of inert pills.

C. Is a progesterone only pill Seasonale is an extended cycle birth control pill containing both levongestrel and estradiol. More spotting occurs in patients taking these pills for the first few months of their use.

All of the following are true of the Transdermal contraceptive patch except: A. Contains ethinyl and estradiol and noregestromin B. Is applied to a fatty area and stays on for 1 week intervals for three consecutive weeks of the month then is kept off for one week to allow for endometrial sloughing. C. Is not as effective as oral contraceptives in preventing pregnancy. D. All of the above

C. Is not as effective as oral contraceptives in preventing pregnancy. The transdermal contraceptive patch is as effective as birth control pills in preventing pregnancy. The advantage of the patch over the pill is that the patient does not have to remember to take a pill at the same time everyday.

A 34-year-old woman complains of fishy-smelling vaginal discharge and is diagnosed to have bacterial vaginosis. What is the most appropriate treatment of her discharge? A. Clindamycin B. Erythromycin C. Metronidazole D. Miconazole E. Doxycycline

C. Metronidazole The treatment of choice for bacterial vaginosis is metronidazole, also an effective treatment for trichomonas vaginitis. The dose is 400 to 500 mg orally twice daily for 1 week. A single dose of 2 g was recently approved (avoid in pregnant woman). Vaginal metronidazole gel or clindamycin cream are also approved forms of treatment. Concurrent therapy of the male partner is controversial. Treatment in pregnancy is recommended because there is a potential association of bacterial vaginosis with preterm labor and delivery.

A 38-year-old female presents with a 10-day history of malodorous vaginal discharge. She denies fevers, chills, pruritus, dysuria, pain during sex, or pelvic pain. She is 24 weeks pregnant and has had an uncomplicated pregnancy to date. She engages in vaginal sexual intercourse with one male partner (her husband), with whom she has been monogamous for 5 years. She is diagnosed with bacterial vaginosis. Which one of the following is a recommended treatment for bacterial vaginosis in a pregnant women? A. Treatment of bacterial vaginosis during pregnancy is not recommended B. Doxycycline 100 mg orally twice a day for 7 days C. Metronidazole gel 0.75%, one full applicator (5 g) intravaginally, once a day for 5 days D. Tinidazole 2 g orally once daily for 2 days E. Treatment should be avoided until after pregnancy

C. Metronidazole gel 0.75%, one full applicator (5 g) intravaginally, once a day for 5 days Treatment of symptomatic bacterial vaginosis is recommended in all pregnant women. Symptomatic bacterial vaginosis in pregnancy has been associated with adverse birth outcomes, including premature rupture of membranes, preterm labor, preterm birth, intra-amniotic infection, and post-partum endometritis in observational studies. In pregnant women, treatment of bacterial vaginosis can reduce the signs and symptoms of vaginal infection, but it is unclear whether treatment of symptomatic (or asymptomatic) bacterial vaginosis reduces the adverse outcomes, particularly preterm birth, associated with bacterial vaginosis in pregnancy.The treatment of symptomatic bacterial vaginosis in pregnant women should be the same as for non-pregnant women—with the same oral or vaginal regimens recommended for non-pregnant women (oral metronidazole or intravaginal metronidazole or clindamycin). Oral therapy has not been shown to be superior to topical therapy in achieving cure rates or preventing adverse outcomes of pregnancy.Although metronidazole crosses the placenta, data suggest that metronidazole use during pregnancy poses low risk of adverse birth outcomes. Multiple cohort and cross-sectional studies of pregnant women with metronidazole exposure have found no evidence of teratogenicity or mutagenic effects in infants. There have been limited studies of the use of tinidazole in pregnant human subjects, but animal data suggest that there is moderate risk with tinidazole use in pregnancy; therefore, it is recommended that tinidazole be avoided during pregnancy.

A twenty six year old female comes into your office to discuss her options for birth control. Her history includes migraines with aura when on combination oral contraceptives in the past. Which would you advise: A. Combination hormone contraceptive pills B. Ortho Evra Patch C. Mirena IUD D. Vaginal Nuvaring

C. Mirena IUD Mirena IUD is the only progesterone only choice. All the other choices contain estrogen. Estrogen can cause migraine headaches with aura.

Patricia has been on oral contraceptives for three months and notes breakthrough bleeding during the third week of pills. You should change her OCP to one that has: A. More estrogenic activity B. Less estrogenic activity C. More progestational activity D. Less progestational activity

C. More progestational activity In women, progesterone levels are relatively low during the preovulatory phase of the menstrual cycle, rise after ovulation, and are elevated during the luteal phase. Progesterone levels tend to be < 2 ng/ml prior to ovulation, and > 5 ng/ml after ovulation. If pregnancy occurs, human chorionic gonadotropin is released maintaining the corpus luteum allowing it to maintain levels of progesterone.

All of the following are correct except: A. Oral contraception failure rate is 3% B. Desogesterol belongs to progesterone family of drugs C. New low dose birth control do not require back up during two weeks D. Oral contraceptives are contraindicated in women over the age of 35 that smoke.

C. New low dose birth control do not require back up during two weeks

A 31-year-old para 2 lady attends the day assessment unit with a history of increased vaginal discharge at 32 weeks' gestation. She is not in labour, and there is no evidence of preterm rupture of the membranes. A high vaginal swab shows bacterial vaginosis. What is the most appropriate initial management? A. Oral erytromycin B. Topical hydrocortisone C. Oral metronidazole D. Clotrimazole pessary E. Reassurance and counseling about hygiene

C. Oral metronidazole

A 22-year-cld female presents with an offensive vaginal discharge History and examination findings are consistent with a diagnosis of bacterial vaginosis. What is the most appropriate initial management? A. Oral azithromycin B. Topical hydrocortisone C. Oral metronidazole D. Clotrimazole pessary E. Advice regarding hygiene and cotton underwear

C. Oral metronidazole Explanation:Bacterial vaginosis (BV) describes an overgrowth of predominately anaerobic organisms such as Gardnerella vaginalis. This leads to a consequent fall in lactic acid producing aerobic lactobaciili resulting in a raised vaginal pH Whilst BV is not a sexually transmitted infection it is seen almost exclusively in sexually active women. Features* Vaginal discharge 'fishy', offensive* Asymptomatic in 50%Amsel's criteria for diagnosis of BV - 3 of the following 4 points should be present1. Thin white homogenous discharge2. Clue cells on microscopy3. Vaginal pH >4.54. Positive whiff test (addition of potassium hydroxide results in fishy odour) Management* Oral metronidazole for 5-7 days* 70-80% initial cure rate* Relapse rate > 50% within 3 months* The BNF suggests topical metronidazole or topical clindamycin as alternatives

An 18-year-old G0 P0 adolescent female is being seen at the physician's office for vaginal discharge. A presumptive diagnosis of bacterial vaginosis is made. Which of the following is a finding consistent with BV? A. pH less than 4.5 B. Frothy vaginal discharge C. Predominance of anaerobes D. Flagellated organisms E. Budding hyphae on KOH examination

C. Predominance of anaerobes

RJ wants to start Depo-Provera injections as her form of birth control. You find no reasons in her history or physical exam that would be contraindications for using this form of birth control. She is currently not using any birth control form. Her LMP was two weeks ago. What instructions should you giver her to initiate her injections? A. She is able to start injections today without further testing B. She is able to start injections today if her UCG is negative C. She must return the first 5 days after her next menstrual cycle to start injections D. None of the above

C. She must return the first 5 days after her next menstrual cycle to start injections

Which of the following is the best description of the relationship between the hemoglobin A1C (glycosylated hemoglobin) level and the glucose concentration of the blood? A. The higher the glycosylated hemoglobin level, the lower the glucose concentration. B. The higher the glucose concentration, the lower the glycosylated hemoglobin level. C. The higher the glucose concentration, the higher the glycosylated hemoglobin level. D. The higher the glycosylated hemoglobin level, the higher the fetal blood glucose level.

C. The higher the glucose concentration, the higher the glycosylated hemoglobin level. The higher the glucose concentration, the higher the glycosylated hemoglobin. Hemoglobin A1c measures the average blood glucose fluctuations over the life span of a maternal RBC, about 120 days. The glycosylated hemoglobin level is not influenced by or related to the fetal blood glucose concentration.

A 37-year-old woman in a monogamous marriage complains of a vaginal discharge with a fishy odor. The vaginal pH is 4.8 and clue cells are present on the smear. A culture reveals presence of Gardnerella vaginalis. The family doctor diagnoses bacterial vaginosis. Each of the following statements is true except for which one. A. This condition may be a cause of premature rupture of membranes in labour B. Bacterial vaginosis may be the cause of premature labour C. The transmission of this condition is sexual D. The condition is associated with postpartum endometritis E. The condition may be a cause of pelvic inflammatory disease (PID)

C. The transmission of this condition is sexual

A 22-year-old G4P1 woman at 26 weeks gestation presents complaining of a postcoital musty odor and increased milky, gray-white discharge for the last week. This was an unplanned pregnancy. She had her first pregnancy at age 15. She reports that she has no new sex partners, but the father of the baby may not be monogamous. On examination, there is a profuse discharge in the vaginal vault, which covers the cervix. Pertinent labs: wet mount pH >4.5 and whiff test positive. Microscopic exam reveals clue cells, but no trichomonads or hyphae. Which of the following is the most appropriate next step in the management of this patient? A. Delay treatment until postpartum B. Treat her now and again during labor C. Treat her now D. Treat her and her partner E. No treatment necessary

C. Treat her now The patient has bacterial vaginosis. All symptomatic pregnant women should be tested and treatment should be not delayed because treatment has reduced the incidence of preterm delivery. The optimal regimen for women during pregnancy is not known, but the oral metronidazole regimens are probably equally effective. Once treated antepartum, there is no need to treat during labor unless she is reinfected.

A 17-year-old female complains of "a bad odor coming from my vagina" after sexual intercourse, moderate vaginal discharge, and vulvar irritation. She is in her first trimester of pregnancy. What is true regarding treatment? A. Azithromycin in first-trimester pregnant women is used to avoid premature rupture of membranes and preterm labor B. Treatment of asymptomatic non pregnant patients is indicated C. Treatment during pregnancy lowers complication rates D. Counsel patient about disulfiram reaction with clindamycin E. Treatment should be avoided until after pregnancy

C. Treatment during pregnancy lowers complication rates

Nelda is breastfeeding her 6-week-old daughter. She comes today with pain, a lump in her right breast, and flulike symptoms. You see that the breast is engorged, erythematous, and warm to the touch. Her temperature is 101.8°F (38.4°C). Your diagnosis is A. breast abscess. B. breast engorgement. C. mastitis. D. viral syndrome.

C. mastitis. With a history of 6 weeks of breastfeeding, pain, and a lump in the breast; flulike symptoms; a temperature of 101.8°F (38.5°C); and physical examination revealing an engorged, erythematous, and warm-to-the-touch breast, your diagnosis is mastitis, an infection of the breast. Mastitis, which occurs in about 5% of lactating women, may be caused by tight clothing, missed infant feedings, poor drainage of the duct and alveolus, or infection with Staphylococcus aureus, Escherichia coli, or Streptococcus. A clogged duct, simple breast engorgement, breast abscess, and viral syndrome are differential diagnoses. A clogged duct and simple breast engorgement may be painful but not erythematous or warm to the touch. Also, flulike symptoms would not be present. A breast abscess should be suspected if there is no resolution of symptoms after several days of antibiotic therapy. If an abscess is present, pitting edema over the affected area is possible. An abscess is usually treated with both antibiotics and drainage.

A 21-year-old female presents with a one-week history of thin, gray vaginal discharge. The discharge began 1 day after her last sexual intercourse when at that time she also noted an unpleasant "fishy" odor. She denies fevers, chills, pain with urination, pelvic pain, pain during sex, or vaginal itching. In the last 3 months she engaged in vaginal sexual intercourse with one new male partner. She has had three male sex partners during the past 12 months and she uses condoms inconsistently. She has no other medical problems and takes no medications. She has never had routine STD screening for Chlamydia trachomatis and Neisseria gonorrhoeae. In addition to ordering routine STD screening tests for Chlamydia trachomatis and Neisseria gonorrhoeae, what are the appropriate next steps for work-up of this woman's symptoms? A. Cervical bacterial culture B. Proline aminopeptidase test card C. pH and KOH testing of vaginal discharge and vaginal discharge microscopy D. Papanicolaou smear E. Polymerase chain reaction assay for Gardnerella vaginalis.

C. pH and KOH testing of vaginal discharge and vaginal discharge microscopy

The leading danger of IUDs and the major cause of morbidity associated with their use is: A. Increased menstrual bleeding B. Dysmenorrhea C. Uterine perforation D. Infection

D. Infection This is why this is not a good method of birth control for women who have multiple sexual partners.

How long can a Nuvaring be out of the vagina before a second form of contraception is necessary? A. 12 hours B. 24 hours C. 1 hour D. 3 hours

D. 3 hours

A 24 year old female asks you about what to do after forgetting to change her patch on the seventh day and she is now on day nine with the same patch on. What advice should you give her? A. If you apply your patch late during the first week then apply a new patch as soon as you remember and make that your new patch day. Use a backup form of birth control for seven days after you apply the new patch. B. If you apply your patch late during week two or three and are one or two days late change it as soon as you remember and utilize a backup form of birth control for seven days C. If you are more than two days late in the second or third week change the patch as soon as you remember and utilize this new day as the day you change your patch. Utilize a secondary form of birth control for seven days.D. All of the above D. All of the above.

D. All of the above. If a patient realizes that her patch has fallen off for less than one day and cannot get the patch to restick to her skin then she should apply a new patch and make that her new patch day. She should use a backup method of birth control for seven days.

A 21-year-old woman who suffers with alcohol dependence attends the gynaecology clinic complaining of a persistent watery vaginal discharge. There is no history of irritation or pruritus. She smokes ten cigarettes a day. On vaginal examination, there is an offensive fishy-smelling vaginal discharge. What is the best treatment for this patient? A. Metronidazole 2 g orally as a single dose B. Metronidazole 500 mg orally twice daily for 14 days C. Clotrimazole vaginal pessary 1 g for 5 nights D. Clindamycin orally 300 mg twice daily for 7 days E. Ceftriaxone 500 mg intramuscularly plus azithromycin 1 g orally

D. Clindamycin orally 300 mg twice daily for 7 days The clinical features are typical of bacterial vaginosis and the other treatments mentioned in the list are not applicable for bacterial vaginosis. The doses of metronidazole are incorrect. Both oral and topical metronidazole should not be used if alcohol cannot be avoided.

A 34-year-old woman is seen in a walk-in clinic for evaluation of vaginal discharge, itching, and discomfort. She has one regular sex partner who is her long-term boyfriend. She is diagnosed with bacterial vaginosis and she prefers not to take an oral antibiotic. You are considering using intravaginal clindamycin for therapy.What is the concern regarding birth control and use of intravaginal clindamycin ovules as treatment for bacterial vaginosis? A. Clindamycin ovules inactivate most spermicides B. Clindamycin penetrates into ovaries and may counteract the desired effect of hormonal contraception to suppress ovulation C. High clindamycin vaginal concentrations enhance sperm motility and prolong sperm survival D. Clindamycin ovules can weaken latex condoms and vaginal diaphragms E. Clindamycin intravaginal gel seems able to decrease cervical mucus quality

D. Clindamycin ovules can weaken latex condoms and vaginal diaphragms Explanation:Use of intravaginal clindamycin for the treatment of bacterial vaginosis can impact the effectiveness of barrier contraception methods. Specifically, clindamycin ovules use an oleaginous base that can weaken latex condoms and other rubber products, such as diaphragms. When a woman receives the 3-day intravaginal clindamycin therapy, she should receive counseling to avoid use of condoms or diaphragm as a means of birth control for at least 72 hours after the last dose of the intravaginal clindamycin

A 34-year-old woman is diagnosed as having a vaginitis based on a "fishy odor" to her vaginal discharge and vaginal pruritus. The cervix is normal in appearance. Microscopic evaluation of a saline "wet prep" of the vaginal secretions reveals squamous epithelial cells coated with coccobacilli along with few white blood cells. Risk factors for developing this condition include all of the following except A. Multiple sexual partners B. Cigarette smoking C. Douching D. Combined oral contraceptive pills E. Lack of vaginal lactobacilli

D. Combined oral contraceptive pills

All of the following interfere with the metabolism of oral contraception except: A. Tetracycline B. Rifampin C. Dylantin/phenytoin D. Corticosteroids

D. Corticosteroids Drugs that effect oral contraception are anticonvulsants, antibiotics, Rifampicin, griseolfulvin, ascorbic acid, and acetominophen

All of the following are mechanical barrier methods of contraception except: A. Diaphragm with spermicidal gel B. The sponge C. Condoms D. Depo Provera injections

D. Depo Provera injections

n regards to depo-provera all of the following are correct except: A. That the first injection must be given during the first five days of menses B. You must first get a pregnancy test before starting the medication C. The second injection must be scheduled for 12 weeks but may be given up to 13 weeks of the last injection D. Depo-Provera is an estrogen based form of contraception given by I.M. injection every three months

D. Depo-Provera is an estrogen based form of contraception given by I.M. injection every three months Depo-Provera is a progesterone only form of contraception

All of the following are key educational inclusions when prescribing an oral contraceptive except: A. What to do when you miss a dose B. Common side effects C. Unusual or concerning side effects D. Dietary restrictions

D. Dietary restrictions

This method of contraception is permanent and requires undergoing a hysteroscopy. A. Tubal ligation B. IUD C. Vasectomy D. Essure

D. Essure ssure: Is a permanent, non-surgical transcervical sterilization procedure for women developed by Conceptus Inc., a subsidiary of Bayer AG. It was approved for use in the United States on November 4, 2002. Hysteroscopy: Small, flexible inserts are placed into the fallopian tubes by a catheter passed from the vagina through the cervix and uterus. The insert contains inner polyethylene terephthalate fibers to induce inflammation causing fibrotic reaction and is held in place by flexible stainless steel inner coil and a dynamic outer nickel titanium alloy coil. Once in place, the device is designed to elicit tissue growth in and around the insert over a period of three months to form an occlusion or blockage in the fallopian tubes; the tissue barrier formed is supposed to prevent sperm from reaching an egg.

Which of these patients needs a cervical cancer screening? A. Lisa, a 45-year-old patient who has atypical squamous cells of uncertain significance (ASCUS) and a human papillomavirus (HPV) positive Pap 1 month ago B. April, a 26-year-old patient who had a negative Pap with negative HPV 1 year ago C. Sondra, a 66-year-old patient who had a negative Pap with negative HPV 11 years, 6 years, and 1 year ago D. Gillian, a 33-year-old patient who had a negative Pap with negative HPV 5 years ago

D. Gillian, a 33-year-old patient who had a negative Pap with negative HPV 5 years ago

Which of the following conditions is contraindicated with utilizing a copper IUD? A. History of ectopic pregnancy B. Nulliparity C. Treated cervical dysplasia D. Heart disease

D. Heart disease

A woman who has experienced bleeding during the first two weeks on birth control pills for three consecutive months should be changed to an OCP that has: A. Lower progestational activity B. Higher progestational activity C. Lower estrogenic activity D. Higher estrogenic activity

D. Higher estrogenic activity

What are the clinical characteristics consistent with a diagnosis of trisomy 21? A. Low birth weight, small jaw with recessed chin, muscle rigidity, and short sternum B. Cleft lip and palate, polydactyly, malformed ears, and absence of the iris C. Lymphedema of the hands and feet, webbed neck, coarctation of the aorta, and urinary tract abnormalities D. Hypotonia, simian creases, epicanthial folds, and Brushfield spots

D. Hypotonia, simian creases, epicanthial folds, and Brushfield spots Characteristics of trisomy 21 (Down syndrome) include hypotonia, simian creases, epicanthial folds, and Brushfield spots. Findings of low birth weight, small jaw and recessed chin, muscle rigidity, and short sternum are consistent with a diagnosis of Edwards' syndrome (trisomy 18). Patau syndrome is indicated by the presence of a cleft lip and palate, polydactyly, malformed ears, and absence of the iris. Clinical findings of lymphedema of the hands and feet, webbed neck, coarctation of the aorta, and urinary tract abnormalities are indicative of Turner's syndrome.

A middle-aged woman is recently diagnosed with Bacterial vaginosis. The following are all true with regard to the diagnostic criteria for Bacterial vaginosis EXCEPT A. It is associated with a cream-coloured discharge B. Clue cells are noted on microscopy C. A fish-like odour is produced on the addition of potassium hydroxide to the discharge D. It is associated with irregular bleeding E. It is associated with a vaginal fluid pH greater than 4.5

D. It is associated with irregular bleeding BV is caused by Gardnerella, Mycoplasma, and overgrowth of anaerobes in the vagina. Women typically present with vaginal discharge which is homogenous, grey/white, thin, watery, copious, and with an offensive fishy smell. Unless associated with another infection such as Candida, it usually does not have other symptoms as itch and soreness. Examination shows discharge coating the vagina and the vestibule and the absence of vaginal inflammation.Amsel's criteria can be used for diagnosis, the presence of 3 out of 4 criteria confirming the diagnosis: 1) vagina pH is >4.5; 2) thin, white homogenous discharge; 3) 'clue cells' (epithelial cells covered in bacteria) seen on wet microscopy; 4) release of fishy odour on adding alkali (10% potassium hydroxide) to drop of discharge on wet mount microscopy.

A 30-year-old housewife complains of a foul smelling vaginal discharge. On examination, discharge was homogenous, fishy odor, and gray in color. What is the most appropriate treatment of her discharge? A. Tetracycline B. Fluconazole C. Cefoxitin D. Metronidazole E. Penicillin

D. Metronidazole Explanation:Treatment options for bacterial vaginosis (BV):1. Oral Metronidazole 500 mg BD for 5-7 days. Treatment of choice.2. Oral Metronidazole 2 g stat.3. Metronidazole vaginal gel 0.75% once daily for 5 days.4. Clindamycin vaginal gel 2% once daily for 7 days.5. Oral tinidazole 2 g stat.6. Oral Clindamycin 300 mg BD for 7 days.(Non of those treatment regimes is known to be superior).

The preferred treatment for a pregnant woman with a positive second trimester screen for bacterial vaginosis is A. Metronidazole gel intravaginally B. Clindamycin orally C. Clindamycin gel intravaginally D. Metronidazole orally E. Tinidazole orally

D. Metronidazole orally

Which of the following medications is an oral estrogen product for women with menopause? A. Provera 2.5 mg B. Estrace 0.01% C. Alora 0.025 mg D. Premarin 0.3 mg

D. Premarin 0.3 mg

Which of the following is a relative contraindication for combo oral contraceptives: A. Undiagnosed vaginal bleeding B. Hepatoma of the liver C. Suspected history of TIAs D. Smoking

D. Smoking Smoking is a relative contraindication. All of the rest of the choices are absolute contraindications.

A 22-year-old woman presents to the GUM clinic with an offensive smelling discharge. She is sexually active and is in a monogamous relationship. She describes no pain or soreness just an offensive smelling discharge. After examination and taking swabs for the second time she is diagnosed with bacterial vaginosis. Which of the following organisms is not likely to be the cause? A. Gardnerella species B. Mobiluncus C. Bacteroides D. Trichomonas E. Mycoplasma

D. Trichomonas Bacterial vaginosis (BV) is not felt to be a sexually transmitted disease but is seen in people who are sexually active. It is a result of an imbalance of the naturally occurring flora in the vagina. It presents as an off-white offensive discharge with a fishy odour. Diagnosis is made with swabs showing clue cells and a loss of vaginal acidity. Trichomonas (D) is a separate infection caused by a protozoon that leads to a greenish discharge. It is a sexually transmitted infection. All the other flora above are responsible for bacterial vaginosis. (A, B, C and E) can all cause BV secondary to the reduction in the normal levels of lactobacilli found in the vagina. This reduction may be due to a recent antibiotic course or a change in pH of the vagina allowing these other bacteria to multiply.

Keeping the pH environment of the vagina in balance helps prevent infection. "pH" is a measure of the acidity of the vaginal environment. In healthy women, vaginal pH is typically 3.8 to 4.5. Which of the following will result in an increase in vaginal pH? A. Bacterial Vaginosis B. Intercourse C. Menopause D. Trichomonasis E. All of the above

E. All of the above

When counseling a woman about the use of oral contraceptives, you should provide her with which of the following pieces of information? A. They do not protect against sexually transmitted infections. B. Antibiotics may decrease their effectiveness and she should use a backup method of protection when taking them C. They must be taken every day. D. They may worsen mood. E. All of the answers are correct.

E. All of the answers are correct.

A 36-year-old G2P2 woman presents to her gynecologist with a 3-week history of vaginal irritation and fish-smelling vaginal discharge. She recently tried an over-the counter antifungal treatment without any improvement in her symptoms. She is sexually active in a monogamous relationship with a male partner of 5 years and she uses a contraceptive ring (NuvaRing). Genitourinary examination shows a thick white discharge. The remainder of her examination is normal. Microscopic evaluation of a saline "wet prep" of the vaginal secretions reveals decreased lactobacilli, a few WBCs, and vaginal epithelial cells with a stippled appearance. Which of the following additional test would support the diagnosis? A. Branching hyphae on 10% KOH prepared microscopic examination B. Positive NAAT for Chlamydia C. Negative gram staining intracellular diplococci D. Vaginal pH less than 4.5 E. Amine odors noted with the addition of 10% KOH

E. Amine odors noted with the addition of 10% KOH

A 26-year-old woman presents for routine gynecologic examination and Pap smear. A thin, homogenous vaginal discharge is noted, and a sample is taken. When potassium hydroxide is added to a wet mount of the sample, a fishy odor is noted. In addition, the Pap smear reveals the presence of "clue cells." Which of the following organisms is likely to be present in increased numbers? A. Staphylococcus aureus B. Neisseria gonorrhoeae C. Candida albicans D. Trichomonas vaginalis E. Gardnerella vaginalis

E. Gardnerella vaginalis

A 30-year-old woman presents with a white, malodorous vaginal discharge. There is no associated itch or dyspareunia. A diagnosis of bacterial vaginosis is suspected. Overgrowth of which one of the following organisms is most likely to cause this presentation? A. Lactobacilli B. Trichomonas C. Candida albicans D. Escherichia coli E. Gardnerella vaginalis

E. Gardnerella vaginalis

A middle-aged woman is recently diagnosed with Bacterial vaginosis. What is the most likely aetiological cause? A. Neisseria gonorrhoeae B. Haemophilus influenzae C. Chlamydia trachomatis D. Staphylococcus aureus E. Gardnerella vaginalis

E. Gardnerella vaginalis BV is a polymicrobial infection characterized by a lack of normal hydrogen peroxide- producing lactobacilli and an overgrowth of anaerobic organisms including G. vaginalis, Mycoplasma hominis, Bacteroides species, Peptostreptococcus species, Fusobacterium species, Prevotella species, and Atopobium vaginae.

A 19-year-old G2P1 African American woman at 30 weeks gestation presents with preterm rupture of membranes six hours ago. Her prenatal course has been complicated by two episodes of bacterial vaginosis for which she was treated. She takes prenatal vitamins and iron. She denies substance abuse or alcohol use, but admits to smoking five cigarettes each day. Her prior pregnancy was delivered vaginally at 41 weeks after spontaneous rupture of membranes. Her blood pressure is 110/70; pulse 84; temperature 98.6°F (37.0°C). Pertinent sonographic findings reveal oligohydramnios and a cervical length of 30 mm. Which of the following is the most likely cause of preterm premature rupture of membranes in this patient? A. Ethnicity B. Smoking C. Previous premature rupture of membranes D. Cervical length E. Genital tract infections

E. Genital tract infections

A female patient is in the emergency department complaining of a malodorous vaginal discharge. The emergency department physician suspects she may have bacterial vaginosis. Which of the following can not help in making the diagnosis? A. Vaginal discharge pH of more than 4.5 B. Clue cells on a microscopy C. Positive whiff test D. Absent leucocytes on a smear E. Green-yellow vaginal discharge

E. Green-yellow vaginal discharge

A 29-year-old sexually active woman presents to the clinic complaining of a malodorous, fishy vaginal discharge for the past 2 weeks. She notices the smell most after sexual activity and does not complain of any pelvic pain. On examination there is a thin, homogeneous, grayish discharge coating her vaginal wall. Vaginal pH is 6.0, and urinalysis for β-human chorionic gonadotropin (β-hCG) is negative. A saline wet mount of the vaginal fluid is shown in the image.What is the mechanism of action of the antibiotic indicated for this condition? A. Binds to the 30S subunit of the ribosome and prevents attachment of aminoacyl- tRNA B. Inhibits DNA-dependent RNA polymerase C. Inhibits ergosterol synthesis D. Inhibits topoisomerase II E. Interacts with DNA to form toxic metabolites in the bacterial cell

E. Interacts with DNA to form toxic metabolites in the bacterial cell Answer (A) is incorrect. Tetracycline antibiotics work by binding to the 30S subunit of the ribosome and preventing attachment of aminoacyl-tRNA. Tetracyclines are also contraindicated in pregnancy. Common toxicities include gastrointestinal distress, discoloration of teeth, inhibition of bone growth in children, and photosensitivity.Answer (B) is incorrect. The antimycobacterial drug rifampin works by inhibiting DNA dependent RNA polymerase. Rifampin toxicities include hepatotoxicity, increasing expression of cytochrome P-450 enzymes and causing bodily fluids to turn red/orange.Answer (C) is incorrect. The azole class of antifungal drugs works by inhibiting ergosterol synthesis. Although vaginal candidiasis is associated with reduced vaginal acidity and malodorous discharge, clue cells are not seen in this condition. Toxicities of azole antifungals include hormone synthesis inhibition, liver dysfunction, fever, and chills.Answer (D) is incorrect. Fluoroquinolone antibiotics work by inhibiting topoisomerase II, also known as DNA gyrase. These drugs are also contraindicated in pregnancy. Common toxicities include gastrointestinal upset, super infections, rash, headache, and dizziness.

Which is true regarding treatment of the partner of the patient with bacterial vaginosis (BV)? A. He should be treated if asymptomatic B. He should be treated if he has balanitis C. Treating the male partner reduces recurrence or reinfection rates D. There is a risk of passing the infection back and forth between partners E. Males often have urethral discharge

E. Males often have urethral discharge Because bacterial vaginosis is not considered to be an STI, partners do not need to be treated, and there is no risk of passing the infection back and forth between partners. A 2016 Cochrane review found high-quality evidence that treating the sexual partners of women with bacterial vaginosis did not affect symptoms, clinical outcomes, or the recurrence of affected women.

A 25-year-old woman presents with an offensive, fishy smelling vaginal discharge. She says it is watery in nature. You suspect that she has bacterial vaginosis (BV). Which of the following is LEAST appropriate for the diagnosis of BV? A. Mixed vaginal flora with few lactobacillus morphotypes on Gram stained vaginal smear B. Clue cells on wet prep C. Vaginal discharge pH of more than 4.5 D. Positive "whiff" test on potassium hydroxide prep E. Positive vaginal culture for Gardnerella vaginalis

E. Positive vaginal culture for Gardnerella vaginalis

In relation to the treatment of BV, which one of the following statements is true? A. Routine treatment of male partners is recommended B. Vaginal douching is beneficial for symptomatic improvement C. Metronidazole is not safe during the first trimester of pregnancy D. Clindamycin 2% intravaginal cream is not effective E. Recurrent BV is difficult to manage as it can recur in up to 70% of females within 3 months of treatment

E. Recurrent BV is difficult to manage as it can recur in up to 70% of females within 3 months of treatment

A 28-year-old lady presents with her partner after two consecutive miscarriages. Her last two miscarriages were in the middle trimester. She complains of a greyish offensive vaginal discharge and her pregnancy test is positive. What is the most appropriate course of action? A. Counseling and offer support B. TORCH (toxoplasmosis, rubella, cytomegalovirus, herpes simplex and human immunodeficiency virus) screening C. Diabetes screeningD D. Commence aspirin and heparin daily throughout pregnancy E. Start oral metronidazole treatment of bacterial vaginosis

E. Start oral metronidazole treatment of bacterial vaginosis

A 37-year-old woman attends your clinic requesting routine gynecologic examination and a cervical smear. A thin, homogenous vaginal discharge is noted, and a sample is taken. When potassium hydroxide is added to a wet mount of the sample, a fishy odor is noted. In addition, her cervical smear reveals the presence of "clue cells." Otherwise the smear was reported as normal. Which is the SINGLE MOST appropriate management? A. Refer for colposcopy B. Repeat the sample in 6 months C. Routine recall in 5 years D. Routine recall in 3 years E. Treat with antibiotics

E. Treat with antibiotics This is bacterial vaginosis so needs treating with metronidazole.

A young female is seen in the women's clinic where it is discovered that she has bacterial vaginosis. All of the following risk factors may have predisposed the patient to this infection except A. Vaginal douching B. An intrauterine device C. Multiple sexual partners over the last year D. Female sexual partner in the last year E. Use of condoms

E. Use of condoms

A 48 year old client with challenging menopausal symptoms is visiting the OB-GYN practice where you practice nursing. She has discussed treatment options with the physician and now has some questions that she would like to further discuss with you. The client includes in her questioning, "What are the potential risks of hormone replacement therapy?" Which of the following is the best answer? a) All options are correct. b) Breast cancer c) Heart disease d) Stroke (CVA)

a) All options are correct.

A nurse is working with a young woman who has been diagnosed with premenstrual syndrome (PMS) to develop a plan to alleviate her symptoms. Which of the following would be most appropriate to include in this plan? a) Engaging in a regular exercise program b) Avoiding the intake of whole grains and fruits c) Decreasing water intake d) Planning work during the days of PMS

a) Engaging in a regular exercise program

A client is taking a progestin-only oral contraceptive, or minipill. When teaching the client about this medication, a nurse should include information on signs and symptoms of: a) tubal or ectopic pregnancy. b) female hypogonadism. c) endometriosis. d) premenstrual syndrome.

a) tubal or ectopic pregnancy. Women taking the minipill have a higher incidence of tubal and ectopic pregnancies, possibly because progestin slows ovum transport through the fallopian tubes. Endometriosis, female hypogonadism, and premenstrual syndrome aren't associated with progestin-only oral contraceptives

A nurse is explaining premenstrual syndrome to a female client. The nurse knows that the client understands when the client states that which of the following appears before menstruation? a) Calmness b) Water loss c) Headache d) Fatigue

c) Headache

A 45-year-old woman is seen because of irregular menstrual periods. Her follicle-stimulating hormone (FSH) level is 48 mIU/mL, and her luteinizing hormone (LH) level is elevated. She asks the clinician what this means. Which would be the best response? a. "You are approaching menopause." b. "You have a hormonal imbalance." c. "Your FSH is normal, but your pituitary is making too much LH." d. "There is an imbalance between yourovaries and pituitary."

a. "You are approaching menopause." FSH: 4.7 to 21.5 mIU/mL*Before puberty - 0 to 4.0 mIU/mL (0 to 4.0 IU/L)*During puberty - 0.3 to 10.0 mIU/mL (0.3 to 10.0 IU/L)*Women who are still menstruating - 4.7 to 21.5 mIU/mL (4.5 to 21.5 IU/L)*After menopause - 25.8 to 134.8 mIU/mL (25.8 to 134.8 IU/L)LH:*women in the follicular phase of the menstrual cycle: 1.9 to 12.5 IU/L*women at the peak of the menstrual cycle: 8.7 to 76.3 IU/L*women in the luteal phase of the menstrual cycle: 0.5 to 16.9 IU/L* pregnant women: less than 1.5 IU/L* women past menopause: 15.9 to 54.0 IU/L*women using contraceptives: 0.7 to 5.6 IU/L

A 36-year-old woman is seen with complaints of vaginal itching, burning, and discharge. On potassium hydroxide (KOH) wet mount of vaginal discharge, the clinician notices hyphae. Which of the following treatments would be appropriate? a. Fluconazole b. Estrogen vaginal cream c. Metronidazole d. Doxycycline

a. Fluconazole This is Candida

A 23-year-old sexually active woman presents for her first Pap smear. Herhistory includes nulligravida, age at first intercourse 14, and more than 10 sexualpartners. Which of the following conditions should the clinician be particularly alert forduring her examination? a. Human papillomavirus (HPV) b. Endometrial hyperplasia c. Vagismus d. Polycystic ovarian syndrome

a. Human papillomavirus (HPV)

A 42-year-old woman presents to the clinic with complaints of painful intercourse for the last month. Which of the following should be explored as the likely cause of her dyspareunia? a. Menopause b. Dehydration c. Excess progesterone d. Sexual trauma as a child

a. Menopause

Which of the following medications is the treatment of choice for trichomonas? a. Metronidazole b. Ceftriaxone c. Diflucan d. Doxycycline

a. Metronidazole

Which of the following would be appropriate treatment for a woman with mild endometriosis? a. Oral contraceptives b. Leuprolide acetate injections c. Nafarelin nasal spray d. Hysterectomy

a. Oral contraceptives

A 32-year-old woman is seen in the clinic because she has been unable to get pregnant after 12 months of unprotected sex. In order to determine the cause of the infertility, the clinician should question her about which of these possible causes? a. Pelvic inflammatory disease b. Oral contraceptive use for 15 years c. Early menarche d. Diet high in soy protein

a. Pelvic inflammatory disease

What is the most common cause of secondary amenorrhea? a. Pregnancy b. Pituitary dysfunction c. Inadequate estrogen levels d. Genetic disorders

a. Pregnancy

Which of the following tests is essential for a 46-year-old woman who the clinician suspects is perimenopausal? a. Pregnancy b. Estrogen level c. Progesterone level d. LH level

a. Pregnancy

A 21-year-old woman is seen in the clinic requesting birth control pills. Which of the following tests is essential before prescribing any oral contraceptive? a. Pregnancy test b. Complete blood cell count c. Thyroid-stimulating hormone d. Urine dip for protein

a. Pregnancy test

A 26-year-old woman is seen with complaints of irregular vaginal bleeding. Which of the following tests should be the first priority? a. Pregnancy test b. Pelvic ultrasound c. Endometrial biopsy d. Platelet count

a. Pregnancy test

A 48-year-old woman is seen in the clinic with complaints of prolonged heavy menstrual periods. She is pale and states she can no longer exercise. Pelvic exam reveals a single, very large mass. Which of the following diagnostic tests should the clinician order first? a. Transvaginal ultrasound b. Endometrial biopsy c. MRI d. Abdominal computed tomography scan

a. Transvaginal ultrasound

A 24 year old female patient who is sexually active c/o vaginal itching. If she has bacterial vaginosis, she might complain of: a. a "fishy" vaginal odor after coitus b. a truncal rash c. frothy bloody vaginal discharge d. midcycle bleeding

a. a "fishy" vaginal odor after coitus

What choice below has no precautions for oral contraceptive pill use? a. gallbladder disease b. 5 months postpartum and lactating c. blood pressure 160/100 d. varicose veins

d. varicose veins

A patient who is 35 years old has identified a small, discrete mass in one breast. How should this be evaluated initially? a. ask whether the mass changes at the time of menses b order a mammogram and ultrasound to assess the mass c. have her return 3-10 days after next menses d. order a mammogram 3-10 days after next menses

a. ask whether the mass changes at the time of menses

After a vaginal exam, a patient received a prescription for metronidazole 500mg twice daily for 7 days. What was her likely diagnosis? a. bacterial vaginosis b. syphilis c. chlamydia d. gonorhea

a. bacterial vaginosis

Young female adult presents with vaginal discharge and itching. Besides trichomoniasis and yeast what else should be included in the differential? a. bacterial vaginosis b. chlamydia c. herpes genitalis d. syphilis

a. bacterial vaginosis

A patient who takes oral contraceptive pills is at increased risk of: a. gallbladder disease b. depression c. hypothyroidism d. varicose veins

a. gallbladder disease

The first step in evaluating a breast lump is: a. history and physical exam b. mammogram c. ultrasound d. MRI

a. history and physical exam

The nurse is discussing the use of hormonal contraception with a woman who just delivered twins and is not ready to get pregnant in the near future. Which methods might the nurse recommend? (Select all that apply.) a) Norplant system b) Intrauterine device c) Oral contraceptives d) Depo-Provera e) Vaginal sponge f) Vaginal ring

b) Intrauterine device c) Oral contraceptives d) Depo-Provera f) Vaginal ring

A client is diagnosed with osteoporosis. Which statements would the nurse include when teaching the client about the disease? Select all that apply. a) Passive ROM exercises can promote bone growth. b) Osteoporosis is a degenerative disease characterized by a decrease in bone density. c) Osteoporosis is common in females after menopause. d) The disease is inherited, caused by an inability to tolerate milk products. e) Osteoporosis can cause pain and injury. f) Weight-bearing exercise would be avoided.

b) Osteoporosis is a degenerative disease characterized by a decrease in bone density. c) Osteoporosis is common in females after menopause. e) Osteoporosis can cause pain and injury

Which of the following dinner selections demonstrates an understanding of nutritional therapy used by women to decrease the signs and symptoms of menopause? a) Corn chips, grapes, lean meat, and baked beans b) Wheat toast, apple slices, broiled chicken breast, and steamed carrots c) Saltine crackers, fruit cocktail, lima beans, and meatloaf d) White toast, apple sauce, grilled chicken, and glazed carrots

b) Wheat toast, apple slices, broiled chicken breast, and steamed carrots To decrease the signs and symptoms of menopause, women are encouraged to decrease their fat and caloric intake and increase their intake of whole grains, fiber, fruit, and vegetables. Saltine crackers, white toast, and corn chips are not good sources of fiber. Fruit cocktail, applesauce, and grapes are high in artificial and natural sugars. Meatloaf is high in fat. Glazed carrots and baked beans can be high in sugar content

A client believes she is experiencing premenstrual syndrome (PMS). The nurse should next ask the client about what symptom? a) menstrual cycle irregularity with increased menstrual flow b) tension and fatigue before menses and through the second day of the menstrual cycle c) midcycle spotting and abdominal pain at the time of ovulation d) mood swings immediately after menses

b) tension and fatigue before menses and through the second day of the menstrual cycle

A 15-year-old girl is seen in the clinic because she has not yet had her first period. Which of the following questions would help the clinician determine the cause? a. "Are you sexually active?" b. "How long have you been underweight?" c. "Did your mother take diethylstilbestrol during her pregnancy?" d. "Have you noticed any changes in your moods lately?"

b. "How long have you been underweight?"

44-year-old patient with breast cancer is prescribed tamoxifen by her surgeon. She is complaining about hot flashes. Which of the following responses by the clinician would be most appropriate? a. "You must be having menopause." b. "The hot flashes are a result of the antiestrogenic effects of tamoxifen." c. "Tamoxifen will impact your temperature regulation center." d. "The drug destroys your ovaries."

b. "The hot flashes are a result of the antiestrogenic effects of tamoxifen."

A 58-year-old woman presents with a breast mass. Which of the following responses by the clinician would be most appropriate? a. "It is probably just a cyst because that is the most common breast mass." b. "We will order a mammogram and ultrasound to help establish a diagnosis." c. "We will go ahead and schedule you for a biopsy because that is the only way to know for sure." d. "Because your lump is painful, it is most likely not cancer."

b. "We will order a mammogram and ultrasound to help establish a diagnosis."

A patient who is scheduled for pelvic exam with PAP sear should be advised to avoid douching, sexual intercourse and tampon use before her exam. For how long should she be advised to avoid these activities for optimal evaluation? a. 24 hours b. 48 hours c. 36 hours d 1 week

b. 48 hours

A 58-year-old woman, who had a total abdominal hysterectomy at the age of 45, is diagnosed with atrophic vaginitis. Which of the following is the most appropriate treatment? a. Conjugated estrogen, 0.625 mg/day oral b. Estradiol, 7.5 mcg/24 hr vaginal ring c. Medroxyprogesterone, 10 mg/day oral d. Conjugated estrogen, 0.3 mg +medroxyprogesterone 1.5 mg/day oral

b. Estradiol, 7.5 mcg/24 hr vaginal ring

A young female has breast buds bilaterally. This represents Tanner Stage: a. I b. II c. III d. IV

b. II

A 17 year old presents with complaints of dysmenorrhea. Which finding below suggests that this is secondary dysmenorrhea? a. normal pelvic exam b. dysmenorrhea is not limited to menses c. unpredictable menses d. nausea with menses

b. dysmenorrhea is not limited to menses

A mammogram in a healthy 50 year old female patient is an example of: a. primary prevention b. secondary prevention c. tertiary prevention d. quaternary prevention

b. secondary prevention

A 20 year old female reports that her grandmother and mother have osteoporosis. What should she be encouraged to do to reduce her risk of osteoporosis? a. aerobic exercise, weight loss, low-fat diet b. smoking cessation, weight bearing exercise c. calcium supplementation, adequate vitamin A and C intake d. optimal caloric intake, vitamin D, supplementation

b. smoking cessation, weight bearing exercise

A 14 year old female has never menstruated. She and her mother are concerned. What is the most important for the NP to assess? a. stature b. tanner stage c. anemia d. family history of amenorrhea

b. tanner stage

During a routine physical examination on a 75-year-old female client, a nurse notes that the client is 5 feet, 3/8 inches (1.6 m) tall. The client states, "How is that possible? I was always 5 feet and 1/2 inches (1.7 m) tall." Which statement is the best response by the nurse? a) "The posture begins to stoop after middle age." b) "After age 40, height may show a gradual decrease as a result of spinal compression" c) "After menopause, the body's bone density declines, resulting in a gradual loss of height." d) "There may be some slight discrepancy between the measuring tools used."

c) "After menopause, the body's bone density declines, resulting in a gradual loss of height."

Which of the following patients in the ED should the advance nurse practitioner treat first? a) A 19-year-old woman with vaginal irritation, malodorous, copious frothy/yellow-green discharge b) A 32-year-old woman with bloating, headache, and reported depression c) A 21-year-old woman reporting sharp colicky abdominal pain, menstrual spotting, and dizziness d) A 48-year-old woman presenting with irregular menses, breast tenderness, and profuse sweating

c) A 21-year-old woman reporting sharp colicky abdominal pain, menstrual spotting, and dizziness The patient with sharp colicky abdominal pain, menstrual spotting, and dizziness has clinical manifestations of an ectopic pregnancy and should be treated first. If untreated, a ruptured ectopic pregnancy can be life threatening. Bloating, headache, and depression are consistent with premenstrual syndrome (PMS) and do not indicate the priority patient at this time. Irregular menses, breast tenderness, and sweating are expected premenopausal symptoms. Vaginal irritation, malodorous copious frothy/yellow-green discharge is consistent with a Trichomonas species infection and is not the priority patient.

he nurse is providing care to a group of clients in an acute care facility. The client most likely to prefer a room that is warm as well as wearing thermal blankets is the client who is a) going through menopause. b) a teenager. c) aged 74 years. d) 6 months pregnant.

c) aged 74 years.

A 40-year-old woman is seen for her yearly examination. She is single and not in a monogamous relationship. Her social history includes smoking cigarettes "occasionally" and drinking about two beers a day. Her body mass index (BMI) is 25. She is requesting birth control. Which of the following methods would be best for this patient? a. Intrauterine device b. Oral contraceptive c. Condom d. Vaginal contraceptive sponge

c. Condom

The recommended time to initiate screening for cervical cancer in women is: a.. prior to becoming sexually active b. at age 18 years c. at age 21 years d. 3 years after first sexual intercourse

c. at age 21 years

A 70 year old female has been in a mutually monogamous relationship for the past 33 years. She has never had an abnormal pap smear. What recommendations should be made regarding pap smears for her? a. they should be continued annually b. they should be preformed every 5 years c. they can be discontinued now d. they can be discontinued at age 75 years

c. they can be discontinued now

Which client statement indicates that the nurse's teaching about oral contraceptive agents has been successful? a) "I can make these drugs more effective by monitoring my basal body temperature." b) "These agents usually only cause a few minor adverse effects when you take them." c) "Oral contraceptives help prevent pregnancy by changing the cervical mucus." d) "If I forget to take my pill one day, I can take 2 pills to get back on schedule"

d) "If I forget to take my pill one day, I can take 2 pills to get back on schedule" If a client forgets to take her oral contraceptive pill, she should take the pill as soon as she remembers. If a whole day has passed, it is okay to take 2 pills on one day to get back on schedule.Oral contraceptive agents may cause numerous and often bothersome or dangerous adverse effects including nausea, vomiting, fluid retention, increased vaginal discharge, chloasma, headaches, weight gain, thromboembolic disorders, hepatic adenoma, and possible hypertension. A history of thromboembolic disease is an absolute contraindication to using oral contraceptive agents.Oral contraceptive agents inhibit ovulation by suppressing follicle-stimulating hormone and luteinizing hormone. They have no effect on cervical mucus.It is not necessary to monitor basal body temperature because ovulation does not occur when the medication is taken properly.

A 55 year old patient has hypertension. She has admitted to her physician that she stopped taking her regular medications about 3 months ago and started an alternative, new-age therapy. When the patient asks about herbal remedies to treat her hot flashes, which of the following natural estrogens is the physician likely to recommend?a) Ginkgo biloba b) Bitter gourd tea c) Turmeric powder d) Soy milk

d) Soy milk Alternative treatments like natural estrogens found in soy products may be effective in treating hot flashes associated with menopause. Ginkgo biloba is an herbal remedy that may help to treat blood disorders and improve memory. The benefits of drinking bitter gourd tea include possible blood sugar regulation, cancer prevention, and antioxidant protection. Turmeric is used in the treatment of digestive and liver problems.

A nurse is reviewing a client's medical history. Which factor indicates the client is at risk for candidiasis? a) Nulliparity b) Use of spermicidal jelly c) Menopause d) Use of corticosteroids

d) Use of corticosteroids

A 51 year old woman has come to the OB/GYN clinic for her annual physical. She tells the nurse that she has been experiencing severe hot flashes, but that she is reluctant to begin hormone therapy (HT). What potential solution should the nurse discuss with the patient? a) Adopting a vegan diet b) Sodium restriction c) Massage therapy d) Vitamin supplements

d) Vitamin supplements

Treatment for mild preeclampsia includes all of the following except: a. Bed rest except for bathroom privileges b. Close monitoring of weight and blood pressure c. Close follow-up of urinary protein, serum creatinine, and platelet count d. A prescription of methyldopa (Aldomet) to control blood pressure

d. A prescription of methyldopa (Aldomet) to control blood pressure Recommended care for women diagnosed with preeclampsia includes bed rest with bathroom privileges, weight and BP monitoring, and closely following urine protein and serum protein, creatinine, and platelet counts. Oral medications are not used as first-line treatment. Codina-Leik, M. T. (2014) Family nurse practitioner certification intensive review fast facts and practice questions

Hot flashes that occur during menopause are thought to be related to: a. Low estrogen levels. b. Low progesterone levels. c. Fluctuating progesterone levels. d. Fluctuating estrogen levels.

d. Fluctuating estrogen levels. Low estrogen levels alone do not produce hot flashes. Female first graders are known to have low estrogen levels but do not have hot flashes. The fluctuation in estrogen levels produces vasomotor symptoms referred to as hot flashes. Hollier, A. (2013) Adult-gero and family nurse practitioner certification practice questions p.73

A 60-year-old woman is seen for an annual checkup. Her obstetric history reveals para 6, gravida 6. She reports that she went through menopause at age 45. Her grandmother died at age 80 of colon cancer, and her father died of lung cancer. What in her history would be a risk factor for ovarian cancer? a. Her numerous pregnancies b. Her age at menopause c. Her father's history of lung cancer d. Her grandmother's history of colon cancer

d. Her grandmother's history of colon cancer

A 50 year old female believes that she is menopausal. She c/o hot flashes and has not had menses in 12 months. Which of the following test results would be expected during menopause? a. increased thyroid-stimulating hormone b. decreased follicle stimulating hormone c. hypoestrogenemia d. increased follicle stimulating hormone

d. increased follicle stimulating hormone

A 52 year old female presents to your clinic with a palpable mass in her right breast. Her last normal mammogram was 6 months ago. What is true about this lump? a. it is probably breast cancer b. it may not be a lump at all c. it is likely a fibroadenoma d. it is probably a benign lesion

d. it is probably a benign lesion

Which type of incontinence would NOT benefit from pelvic floor muscle training? a. stress b. urge c. mixed d. overflow

d. overflow

A female patient c/o dysuria with vaginal discharge. How should this be managed? a. order a urinalysis b. order a urine culture c. perform a pelvic exam d. perform an abdominal exam, urinalysis and pelvic exam

d. perform an abdominal exam, urinalysis and pelvic exam

A female patient who takes oral contraceptives has just completed her morning exercise routine. She c/o pain in her right calf. Her BP and HR are normal. She is not shout of breath. Her calf is red and warm to touch. What is NOT a part of the differential diagnosis? a. Deep vein thrombosis b. cellulitis c. calf muscle strain d. sciatica

d. sciatica


Conjuntos de estudio relacionados

blood transfusions complications /Hematology/ DIC/ITP/HIT

View Set

Apex Learning Quiz 7.1.4 Review CCR

View Set

FINN 3279 Poll Questions and Quizzes

View Set

Unreal Engine/C++ Interview Questions

View Set

Astronomy Final Exam Concept Quizzes

View Set

MKT 291 FINAL EXAM - PRACTICE PROBLEMS

View Set

Vertical-Mast Sit-Down Counterbalance Forklifts: Module 22107

View Set

Ch. 1 - Introduction to Photovoltaic Systems

View Set